44 46 51 52 53 56

¡Supera tus tareas y exámenes ahora con Quizwiz!

21) The nurse is assessing a pediatric client who is experiencing metabolic acidosis. Which assessment questions should the nurse include when interviewing the child's parents? Select all that apply. 1. "Has your child consumed any aspirin?" 2. "Has your child consumed any boric acid?" 3. "Has your child consumed any antifreeze?" 4. "Has your child consumed any baking soda?" 5. "Has your child consumed any antacids?

" Answer: 1, 2, 3 Explanation: 1. An overdose of aspirin is associated with metabolic acidosis. 2. Consumption of boric acid can cause metabolic acidosis. 3. Accidental consumption of antifreeze can cause metabolic acidosis. 4. Consumption of baking soda is associated with metabolic alkalosis. 5. Consumption of antacids is associated with metabolic alkalosis.

The classic clinical triad of intussusception is intermittent, severe, crampy ____ ____; a palpable sausage-shaped mass on the right side of the _____; and currant ____ ____.

abdominal pain, abdome, jelly stools

3) Which parental statement indicates correct understanding of preventive techniques for heat-related illnesses when children exercise? 1. "Wearing dark clothing during exercise is recommended." 2. "Water is the fluid of choice to replenish fluids." 3. "During activity, stop for fluids every 15 to 20 minutes." 4. "Hydration should occur at the end of an exercise session.

" Answer: 3 Explanation: 1. Light-colored, light clothing is best to wear during exercise activities; wearing of dark colors can increase sweating. 2. A combination of water and sports drinks is best to replace fluids during exercise. 3. During activity, stopping for fluids every 15 to 20 minutes is recommended. 4. Hydration should occur before and during the activity, not just at the end.

22) The nurse is assessing a pediatric client who is experiencing metabolic alkalosis. Which assessment questions should the nurse include when interviewing the child's parents? Select all that apply. 1. "Has your child consumed any aspirin?" 2. "Has your child consumed any boric acid?" 3. "Has your child consumed any antifreeze?" 4. "Has your child consumed any baking soda?" 5. "Has your child consumed any antacids?

" Answer: 4, 5 Explanation: 1. An overdose of aspirin is associated with metabolic acidosis. 2. Consumption of boric acid can cause metabolic acidosis. 3. Accidental consumption of antifreeze can cause metabolic acidosis. 4. Consumption of baking soda is associated with metabolic alkalosis. 5. Consumption of antacids is associated with metabolic alkalosis.

The nurse is caring for a school-age child who has chronic fluid overload with edema, and teaches the parents about skin care for their child. Which statement by the parents indicates the need to review the material further? a. "Pajamas sound ideal for clothes." b. "Places where the skin rubs together are risk areas for breakdown." c. "It is best to buy clothes that are loose-fitting, so they do not rub the skin." d. "We should check the skin daily to look for any red areas.

" Answer: a. "Pajamas sound ideal for clothes." Feedback: The statement "Pajamas sound ideal for clothes" indicates further review is necessary. Dressing the child in pajamas promotes the sick role, singles the child out, and does not promote self-esteem. Body image is a concern with edema. The remaining statements are appropriate.

The neonatal nurse explains to new parents that infants are at greater risk for fluid and electrolyte imbalance than older children are. Which parent comment would indicate that further education is needed? a. "Compared to an adult, an infant has little body water for reserve." b. "Infants maintain their temperature by losing heat through their heads." c. "Infants have a higher metabolic rate than older children do." d. "Infants lose water through their skin, and they have a larger proportion of skin surface area than older children do.

" Answer: b. "Infants maintain their temperature by losing heat through their heads." Feedback: Incorrect answers indicate the parents understand the lesson taught, and do not require further education. A correct answer indicates the parents do not understand the lesson, and require further education. Losing heat through their heads will have minimal affect on fluid loss in infants. A parent who makes this comment will require further education.

A nurse obtains a history from a single, breastfeeding mother with a small but hydrated 3-month-old infant who is listless following what the mother describes as a seizure. Which question would be most important for the nurse to ask? a. "Did you have gestational diabetes during pregnancy?" b. "Is this your first baby?" c. "Are you sure you didn't hurt the baby?" d. "Has your baby had seizures before?

" Answer: d. "Has your baby had seizures before?" Feedback: "Are you sure you didn't hurt the baby?" is judgmental and closed-ended. Questions should be open-ended. The nurse should be nonjudgmental in attitude and expressions.

The home health nurse is taking care of a 5-year-old with mumps and teaches his mother about his care. Which statement by the parent indicates understanding? a. "I have to keep my son out of school until the swelling subsides." b. "He can have all the orange juice he wants to keep him hydrated." c. "He will be contagious as long as his parotids are swollen." d. "I will watch him closely because he might contract viral meningitis.

" Answer: d. "I will watch him closely because he might contract viral meningitis." Feedback: Viral meningitis is rare, but may occur. The parent should be alert for complications. Keep children out of school or child care until 5 days after parotid swelling occurs. Avoid foods and beverages that increase salivary flow and cause pain (e.g., citrus, spices, and candies). The child is contagious up to 5 days before and after the onset of parotid swelling.

6) The nurse is providing care to a child who is diagnosed with Legg-Calvé-Perthes disease. Which parental statement regarding the child's care required further teaching from the nurse? 1. "We're glad this will only take about 6 weeks to correct." 2. "We understand abduction of the affected leg is important." 3. "We know to watch for areas on the skin that the brace might rub." 4. "We understand swimming is a good sport for Legg-Calvé-Perthes.

"Answer: 1 Explanation: 1. The treatment for Legg-Calvé-Perthes disease takes approximately 2 years. 2. The leg should be kept in the abducted position to prevent damage to the head of the femur due to Legg-Calvé-Perthes disease. 3. A brace is a component of the treatment of Legg-Calvé-Perthes disease and is worn to prevent damage to the head of the femur, so skin irritation should be monitored. 4. Swimming is a good activity to increase mobility in a child with Legg-Calvé-Perthes disease.

15) Which statements should the nurse include in the discharge medication teaching for a child diagnosed with asthma who is prescribed cromolyn sodium (a mast cell stabilizer)? Select all that apply. 1. "The medication works to prevent exacerbations." 2. "The medication should be administered at the first symptom of an asthmatic attack." 3. "The medication should be taken on a daily basis." 4. "The medication should not be administered if the child has a cold." 5. "The medication desensitizes the child against specific allergens.

"Answer: 1, 3 Explanation: 1. This statement is true. Cromolyn sodium is used to inhibit an asthmatic response to allergens. 2. This is incorrect. This medication does not improve the child's condition during an asthmatic attack. 3. This is a preventative medication so doses should not be missed. 4. The medication should be taken daily. 5. This medication does not desensitize the child against allergens.

6) The nurse receives a phone call from the parent of a child who is prescribed rifampin (Rimactane) for treatment of tuberculosis because she saw that the child's urine was orange. Which response by the nurse is accurate? 1. "Encourage your child to drink cranberry juice." 2. "An orange discoloration of urine is expected while your child is on this medication." 3. "Bring your child to the clinic for a urinalysis." 4. "Bring your child to the clinic for a radiograph of the kidneys.

"Answer: 2 Explanation: 1. Orange urine does not mean the child has a urinary tract infection, and a urinalysis, radiograph, and encouragement of cranberry juice would not be options. 2. Rifampin can color the urine orange, so the parents and child should be taught that this is an expected side effect. 3. Orange urine does not mean the child has a urinary tract infection, and a urinalysis, radiograph, and encouragement of cranberry juice would not be options. 4. Orange urine does not mean the child has a urinary tract infection, and a urinalysis, radiograph, and encouragement of cranberry juice would not be options.

1) The mother of a toddler-age client states, "My daughter seems to be at an increased risk for complications associated with respiratory infections." Which response by the nurse is accurate? 1. "You are incorrect in your assessment." 2. "The younger child's airways are smaller and more easily occluded." 3. "Air passages are more likely to become blocked with mucus because younger children make more mucus than older children." 4. "Toddlers do not breathe as deeply as do older children.

"Answer: 2 Explanation: 1. The mother is correct in her statement. 2. Airways are smaller in the younger child and are more easily occluded when mucus is produced. 3. Blockage of air passages with mucus is not related to the age of the child but more to the etiology of mucus production and the continuation of the causative agent. 4. Depth of breathing is not age dependent.

9) Which parental statement indicates correct understanding regarding pancreatic enzyme administration in the treatment of cystic fibrosis? 1. "I will administer this medication 4 times each day." 2. "I will administer this medication twice each day." 3. "I will administer this medication with meals and snacks." 4. "I will administer this medication every 6 hours around the clock.

"Answer: 3 Explanation: 1. A scheduled time would not be appropriate because the enzymes are used to assist in digestion of nutrients. 2. A scheduled time would not be appropriate because the enzymes are used to assist in digestion of nutrients. 3. Pancreatic enzymes are administered with meals and large snacks. 4. A scheduled time would not be appropriate because the enzymes are used to assist in digestion of nutrients.

7) The nurse is giving discharge instructions to the parents of a child whose adrenal glands have been removed due to a tumor. Which parental statement indicates the need for further education? 1. "I will call the doctor if my child has restlessness and confusion." 2. "If my child has any gastric irritation, I will give him antacids." 3. "If my child has vomiting and diarrhea, I will hold his hydrocortisone." 4. "I will give my child his hydrocortisone in the morning.

"Answer: 3 Explanation: 1. Restlessness and confusion can be signs of adrenal insufficiency, and the healthcare provider would need to know these symptoms. No further instruction is needed. 2. Hydrocortisone can cause gastric irritation, and antacids are given between meals. No further instruction is needed. 3. If the child is ill and cannot take hydrocortisone by mouth, the child would need to have an injection. Failure to give hydrocortisone could lead to severe illness and cardiovascular collapse. The mother needs additional instruction. 4. The child should have hydrocortisone in the morning, which mimics the normal diurnal pattern of cortisol secretion. No further instruction is needed.

3) Which parental statement would cause the nurse to include further education related to the care required for a child who is diagnosed with congenital clubfoot? 1. "We're getting a special car seat to accommodate the casts." 2. "We'll watch for any swelling of the feet while the casts are on." 3. "We'll keep the casts dry." 4. "We're happy this is the only cast our baby will need.

"Answer: 4 Explanation: 1. Using a car seat is the law. Special car seats to accommodate the casts are available and should be utilized. 2. Parents should be watching for swelling while the casts are on. 3. Keeping the casts dry is important to prevent complications. 4. Serial casting is the treatment of choice for congenital clubfoot. The cast is changed every 1 to 2 weeks until the corrected foot position is achieved.

7) Which parental statement at the conclusion of a teaching session regarding environmental controls for childhood asthma indicates correct understanding of the information presented? 1. "We're glad the dog can continue to sleep in our child's room." 2. "We'll keep the plants in our child's room dusted." 3. "We'll be sure to use the fireplace often to keep the house warm in the winter." 4. "We will replace the carpet in our child's bedroom with tile.

"Answer: 4 Explanation: 1. When possible, pets and plants should not be kept in the home. 2. When possible, pets and plants should not be kept in the home. 3. Smoke from fireplaces should be eliminated. 4. Control of dust in the child's bedroom is an important aspect of environmental control for asthma management.

The nurse is teaching a prenatal class about respiratory infections. Which statement by a parent indicates that further teaching is necessary? a. "When my newborn has a stuffy nose, he will be okay because newborns are obligatory mouth breathers." b. "Children's narrower airways cause them to breathe harder when they are congested." c. "The only time a newborn breathes through the mouth is when he's crying." d. "I should keep my newborn's nose clean so he can breathe and eat without difficulty.

"Answer: a. "When my newborn has a stuffy nose, he will be okay because newborns are obligatory mouth breathers." Feedback: Newborns are obligatory nose breathers. The only time newborns breathe through the mouth is when they are crying. The other statements by the parent are correct.

Which statement by a 17-year-old girl indicates the need for additional counseling regarding the use of medications for TB treatment? a. Isoniazid: "I should take this when I eat." b. Rifampin: "My contact lenses will turn orange." c. Isoniazid: "No more drinking parties for me." d. Rifampin: "I need to stop taking my birth control pills.

"Answer: a. Isoniazid: "I should take this when I eat." Feedback: Isoniazid should be taken 1 hour before or 2 hours after meals. All of the other statements are correct.

An 8-year-old child is diagnosed with viral pneumonia and sent home from the clinic without an antibiotic prescription. The symptoms worsen, and the child returns to the clinic a week later with signs of a higher fever, listlessness, and a harsh, productive cough. The child's mother states, "I knew a prescription for antibiotics was needed." Which response by the nurse is the most appropriate? a. "It is better to wait to make sure so we don't use antibiotics unnecessarily. This approach also saves healthcare dollars." b. "Antibiotics are not effective for viral pneumonia. Bacteria can grow later in the course of the illness, requiring the need for antibiotics at that time." c. "You do not want to expose your child to medication unnecessarily. Now it is necessary, because it is bacterial pneumonia." d. "Sometimes we just do not know. I'm glad you came back in.

"Answer: b. "Antibiotics are not effective for viral pneumonia. Bacteria can grow later in the course of the illness, requiring the need for antibiotics at that time." Feedback: The nurse responds with the most informative, accurate response. The decision not to use antibiotics for viral pneumonia was based on sound rationale about the etiology of the illness, not cost.

A nurse explains why a 4-year-old presenting with respiratory distress has retractions. Which statement by the parent indicates that the teaching was understood? a. "When distress occurs, children swallow air, leading to expansion of the rib cage and retractions." b. "Retractions occur in all children, because their ribs are soft and pliable. They are not related to respiratory distress." c. "Children breathe primarily with their diaphragm, but when distress occurs, the muscles between the rib cage work with extra effort to move air through narrow airways." d. "Children breathe primarily with the muscles between the ribs, so when distress occurs, the extra work of breathing causes retractions.

"Answer: c. "Children breathe primarily with their diaphragm, but when distress occurs, the muscles between the rib cage work with extra effort to move air through narrow airways." Feedback: Up to the age of 6 years, children breathe primarily with their diaphragm. The intercostal muscles assist by increasing the chest diameter. When distress occurs, the intercostal muscles of the rib cage work with extra effort to move air through narrow airways. This causes retractions.

2) Which nursing diagnosis should the nurse include in the plan of care for an infant diagnosed with acute bronchiolitis due to respiratory syncytial virus (RSV)? 1. Activity Intolerance 2. Ineffective Peripheral Tissue Perfusion 3. Acute Pain 4. Decreased Cardiac Output

Answer: 1 Explanation: 1. Activity Intolerance is a problem because of the imbalance between oxygen supply and demand. 2. Tissue perfusion (peripheral) is not affected by this respiratory disease process. 3. Acute Pain is not usually associated with acute bronchiolitis. 4. Cardiac Output is not compromised during an acute phase of bronchiolitis.

10) The nurse is expecting the admission of a child with severe isotonic dehydration. Which intravenous fluid prescription does the nurse anticipate for this child? 1. 0.9% normal saline (NS) 2. D5 0.2% (¼) normal saline 3. D5W 4. Albumin

Answer: 1 Explanation: 1. 0.9% normal saline (NS) is an isotonic fluid and maintains Na and chloride at present levels. 2. D5 0.2% (¼) normal saline would not be used initially but later, as maintenance fluids. 3. D5W can lower sodium levels, and so it would not be used to replace fluids in severe isotonic dehydration. 4. Albumin is used to restore plasma proteins.

7) A nurse is planning care for a child with hyponatremia. The nurse, delegating care of this child to a licensed vocational nurse (LVN), cautions the LVN to immediately report which clinical manifestation? 1. Seizures 2. Respiratory distress 3. Hyperthermia 4. Bradycardia

Answer: 1 Explanation: 1. A child with hyponatremia is at risk for seizures. 2. Respiratory distress is not a risk of hyponatremia. 3. Hyperthermia is not a risk of hyponatremia. 4. Bradycardia is not a risk of hyponatremia.

5) In the morning, a nurse receives change-of-shift report on four pediatric clients, each of whom has some form of fluid-volume excess. Which of the children should the nurse see first? 1. The child with tachypnea and pulmonary congestion 2. The child with hepatomegaly and normal respiratory rate 3. The child with dependent and sacral edema and regular pulse 4. The child with periorbital edema and normal respiratory rate

Answer: 1 Explanation: 1. A child with respiratory distress should be the first client the nurse checks after receiving a report. 2. The child with hepatomegaly and normal respiratory rate is more stable than the child with tachypnea and pulmonary congestion. 3. The child with dependent and sacral edema and regular pulse is more stable than the child with tachypnea and pulmonary congestion. 4. The child with periorbital edema and normal respiratory rate is more stable than the child with tachypnea and pulmonary congestion.

8) Which menu choices for a child who is diagnosed with renal failure and experiencing hyperkalemia indicate the need for further instruction by the nurse? 1. Carrots and green, leafy vegetables 2. Spaghetti and meat sauce with breadsticks 3. Hamburger on a bun and cherry gelatin 4. Chips, cold cuts, and canned foods

Answer: 1 Explanation: 1. Carrots and green, leafy vegetables are high in potassium. 2. Spaghetti and meat sauce with breadsticks would be acceptable choices for a low-potassium diet. 3. Hamburger on a bun with cherry gelatin would be acceptable choices for a low-potassium diet. 4. Chips, cold cuts, and canned foods are high in sodium but not necessarily in potassium.

7) The nurse is preparing medication instruction for a child who has undergone a kidney transplant and is prescribed cyclosporine. The parents ask the nurse about the reason for the cyclosporine. Which rationale for this medication should the nurse include in the response? 1. Suppress rejection 2. Decrease pain 3. Improve circulation 4. Boost immunity

Answer: 1 Explanation: 1. Cyclosporine is given to suppress rejection. 2. Cyclosporine does not decrease pain. 3. Cyclosporine does not affect circulation. 4. Cyclosporine does not boost immunity.

16) As a result of opioid administration, a child's respirations are slow and shallow. Which should the nurse anticipate when assessing the child's arterial blood gas? 1. Increased PCO2 and respiratory acidosis 2. Decreased PCO2 and respiratory alkalosis 3. Low pH and low PCO2 4. High pH and high PCO2

Answer: 1 Explanation: 1. Due to inadequate respirations, the child retains CO2 and develops respiratory acidosis. 2. This statement is incorrect as the child retains carbon dioxide. 3. The pH would be acidic, but the pH would be high. 4. The child would have a low pH (acidosis) and high PCO2.

12) A child is admitted to the hospital for hypercalcemia and is placed on diuretic therapy. Which diuretic would the nurse expect to give? 1. Furosemide (Lasix) 2. Hydrochlorothiazide (Aquazide) 3. Spironolactone (Aldactone) 4. Mannitol (Osmitrol)

Answer: 1 Explanation: 1. Furosemide (Lasix) is the diuretic used to aid in excretion of calcium. 2. Thiazide diuretics (like hydrochlorothiazide) decrease calcium excretion and should not be given to the hypercalcemic client. 3. Spironolactone (Aldactone) is a potassium-sparing diuretic. While there is a net increase in calcium in the urine, it is not as effective an option as furosemide. 4. Mannitol (Osmitrol) is a diuretic used to decrease cerebral edema and is not routinely used to aid in excretion of calcium.

4) A 5-year-old child with a history of hypopituitarism presents with complaints of right hip and leg pain. Which prescribed medication for the diagnosis should the nurse identify as the cause for the current symptoms? 1. Daily growth hormone 2. Insulin before meals and bedtime 3. DDAVP (desmopressin) at bedtime 4. Cortisone injections

Answer: 1 Explanation: 1. Growth hormone injections and hypopituitarism have been associated with slipped capital femoral epiphysis, which manifests with complaints of hip or knee pain. 2. Insulin is not used to treat hypopituitarism. 3. DDAVP reduces urinary output. It does not cause slipped capital femoral epiphysis. 4. Cortisone is not used for hypopituitarism.

16) Which is the priority nursing action when preparing a neonate born with a gastroschisis defect for transport to a pediatric hospital for corrective surgery? 1. Covering the exposed intestines with sterile moist gauze 2. Wrapping the newborn warmly in two or three blankets 3. Providing a sterile water feeding to maintain hydration during transport 4. Allowing the parents of the newborn to see their child prior to transport

Answer: 1 Explanation: 1. It is important to keep the intestine from drying during transport. 2. Placement in a transport isolette would be preferred to wrapping due to the nature of the birth defect. 3. The newborn should be NPO. 4. While it is important for the parents to see their child before transport, this is not the priority nursing intervention.

11) Which action related to insulin administration should the nurse include in the teaching plan for an adolescent client who has been newly diagnosed with diabetes mellitus to avoid the development of lipoatrophy? 1. Rotating injection sites 2. Checking blood sugars at mealtime and bedtime 3. Using a sliding scale for additional coverage 4. Administration of insulin via insulin pump

Answer: 1 Explanation: 1. Lipoatrophy is caused by using the same insulin injection site. 2. Checking blood sugars does not influence lipoatrophy. 3. A sliding scale does not influence lipoatrophy. 4. Insulin administration via pump does not influence lipoatrophy.

1) Which clinical manifestations should the nurse anticipate when assessing a child who has been admitted to the hospital unit with a diagnosis of minimal change nephrotic syndrome (MCNS)? 1. Massive proteinuria, hypoalbuminemia, and edema 2. Hematuria, bacteriuria, and weight gain 3. Urine specific gravity decreased and urinary output increased 4. Gross hematuria, albuminuria, and fever

Answer: 1 Explanation: 1. Nephrotic syndrome is an alteration in kidney function secondary to increased glomerular basement membrane permeability to plasma protein. It is characterized by massive proteinuria, hypoalbuminemia, and edema. 2. Bacteriuria and fever are associated with a urinary tract infection. Because of the edema, a weight gain, not a weight loss, would be seen. 3. In MCNS, the urine output decreases and the specific gravity of urine increases. 4. Gross hematuria and hypertension are associated with glomerulonephritis.

6) A newborn diagnosed with an omphalocele defect is admitted to the intensive care nursery. Which nursing action is appropriate based on the current data? 1. Placing the newborn on a radiant warmer 2. Placing the newborn in an open crib 3. Preparing the newborn for phototherapy 4. Preparing the newborn for a bottlefeeding

Answer: 1 Explanation: 1. Omphalocele is a congenital malformation in which intra-abdominal contents herniate through the umbilical cord. The newborn loses heat through the viscera; a warmer is indicated to prevent hypothermia. 2. The crib would not provide adequate maintenance of temperature control. 3. Phototherapy is used to treat hyperbilirubinemia, not an omphalocele defect. 4. The newborn will require surgical correction of the defect prior to initiating bottle or breast feeding.

11) A 6-year-old child is hypokalemic. Which menu choice should the nurse encourage for this child? 1. Pizza with a fruit plate 2. Chicken strips with chips 3. Fajita with rice 4. A hamburger with French fries

Answer: 1 Explanation: 1. Pizza with a fruit plate should be encouraged because fruits (bananas, apricots, cantaloupe, cherries, peaches, and strawberries) have high amounts of potassium, and a child is likely to eat this combination. 2. Chicken strips and chips are not good sources of potassium. 3. The nurse is looking for potassium-rich foods that are attractive to children. This choice does not meet the requirement. 4. A hamburger and French fries do not provide potassium.

6) A hospitalized child has been diagnosed with SIADH (syndrome of inappropriate antidiuretic hormone), a complication of his meningitis. Which laboratory data should the nurse anticipate for this child? 1. Hyponatremia 2. Hypocalcemia 3. Hyperglycemia 4. Hypernatremia

Answer: 1 Explanation: 1. SIADH is associated with increased permeability in distal renal tubes, leading to water intoxication and low sodium. 2. Hypocalcemia is not seen with SIADH. 3. Hyperglycemia is not related to SIADH. 4. Hypernatremia is seen with diabetes insipidus, not SIADH.

10) A child returns from exploratory surgery following a gunshot wound to the abdomen. Which nursing intervention should be excluded for the plan of care? 1. Immediate initiation of oral feedings 2. Assessment of the surgical site 3. Administration of opioid narcotics for pain management 4. Visitation at the bedside

Answer: 1 Explanation: 1. The child will be NPO after an exploratory abdominal surgery. The nurse should exclude this from the child's plan of care. 2. The surgical site must be visualized frequently for bleeding. 3. Pain management is essential and opioid analgesics are often necessary after exploratory surgery. 4. This describes family-centered care; parents should be involved as much as possible and should be present before the child wakes up.

4) A preschool-age child is admitted to the hospital with acute postinfectious glomerulonephritis (APIGN) and is admitted to the hospital. Which is the priority nursing diagnosis for this child? 1. Risk for Injury related to hypertension. 2. Altered Growth and Development related to a chronic disease. 3. Risk for Infection related to hypertension. 4. Fluid Volume Excess related to decreased plasma filtration

Answer: 1 Explanation: 1. The child with APIGN has marked hypertension, which can lead to cardiac failure and cerebral injuries. 2. Growth and development are not normally affected because this is an acute process, not a chronic one. 3. While a risk for infection might be present, it is not related to the hypertension. 4. Although fluid retention occurs, this is not the priority diagnosis.

6) The nurse is caring for a child on bed rest who has severe edema in a left lower extremity due to blocked lymphatic drainage. Which nursing diagnosis would take priority? 1. Risk for Impaired Skin Integrity 2. Risk for Altered Body Image 3. Risk for Imbalanced Nutrition: Less Than Body Requirements 4. Risk for Activity Intolerance

Answer: 1 Explanation: 1. The highest priority problem is skin integrity. 2. Body image would not take priority over the integrity of the skin for this scenario. 3. Nutrition would not take priority over the integrity of the skin for this scenario. 4. Activity intolerance would not take priority over the integrity of the skin for this scenario.

16) Which assessment finding would cause the nurse to question whether a preschool-age boy, diagnosed with phenylketonuria shortly after birth, is following the prescribed dietary restrictions? 1. The child's body has a musty odor. 2. This child is a blue-eyed blond. 3. The child appears sleepy and uninterested in the surroundings. 4. The child has a sunburn over his entire body.

Answer: 1 Explanation: 1. The odor is caused by the excretion of phenylketone by-products through the skin and would indicate noncompliance with the dietary restrictions. 2. While this is a characteristic of most children with PKU, it is not related to dietary intake or restrictions. 3. This is not a symptom of untreated phenylketonuria. 4. Photophobia is not associated with PKU. The child with untreated PKU has an eczematous rash. Page Ref: 1428

13) Which independent nursing action is appropriate for a 2-month-old infant who is a direct admission to the pediatric unit with a diagnosis of ALTE (apparent life-threatening event)? 1. Place the child on an apnea monitor. 2. Place the child on nasal cannula oxygen. 3. Draw blood for arterial blood gases. 4. Place the child on contact isolation.

Answer: 1 Explanation: 1. This is appropriate monitoring of the infant. 2. Oxygen is a dependent order except under emergency situations. There is no evidence the child needs oxygen. 3. Laboratory tests are not an independent action. 4. There is no indication of a respiratory infection. At this time, contact isolation is not indicated.

12) The nurse is unsuccessful in inserting a nasogastric tube for a newborn client. The nurse suspects the newborn has esophageal atresia / tracheoesophageal (EA/TE) fistula. Which nursing action is appropriate while waiting for the healthcare provider to further assess the neonate? 1. Position the newborn in semi-Fowler position. 2. Allow the newborn to be taken to the mother's room for bonding. 3. Offer the newborn formula feeding instead of breastfeeding. 4. Wrap the newborn in blankets and place in a crib by the viewing window.

Answer: 1 Explanation: 1. This will reduce stomach juices from being aspirated into the lungs. 2. Because an anomaly is suspected, the newborn should remain under visualization until the diagnosis is confirmed and medical orders determined. 3. If an EA/TE fistula is suspected, the feeding should be withheld until the diagnosis is confirmed or cleared. 4. A newborn wrapped in blankets cannot be observed clearly. The child should be placed in an over-bed warmer.

3) A toddler-age client presents to the emergency department with a sore throat and difficulty swallowing. The nurse suspects acute epiglottitis. Which nursing action is avoided based on the current assessment data? 1. Throat culture 2. Medical history 3. Vital signs 4. Auscultation of breath sounds

Answer: 1 Explanation: 1. Throat cultures should never be done when a diagnosis of epiglottis is suspected. Manipulation of the throat can stimulate the gag reflex in an already inflamed airway and can cause complete occlusion of the airway. 2. Medical history should be obtained, which assists in diagnosis. 3. Vital signs should always be taken when assessment is done. 4. Assessment of breath sounds is essential for diagnosis.

21) Which are the leading causes of pediatric abdominal injuries for which the nurse should provide client teaching during scheduled health maintenance visits? Select all that apply. 1. Motor vehicle crashes 2. Falls 3. Blunt trauma 4. Stabbing 5. Impalement

Answer: 1, 2, 3 Explanation: 1. Motor vehicle crashes are a leading cause of pediatric abdominal injuries. The nurse should provide education related to proper use of seat belts during health maintenance visits to decrease the incidence of abdominal injuries. 2. Falls are a leading cause of pediatric abdominal injuries. The nurse should include education regarding age-appropriate pediatric fall prevention during health maintenance visits. 3. Blunt trauma is a leading cause of pediatric abdominal injuries. The nurse should include prevention strategies during health maintenance visits. 4. While stabbing can cause abdominal injury, this is not a common cause in the pediatric population. 5. While impalement can cause abdominal injury, this is not a common cause in the pediatric population.

20) Which nutritional interventions should the nurse include in the plan of care for a pediatric client who is receiving peritoneal dialysis in the treatment of chronic renal failure? Select all that apply. 1. Provide small, frequent meals. 2. Avoid battles over nutritional intake. 3. Administer supplements by tube feedings, if needed. 4. Implement hand hygiene frequently. 5. Perform daily catheter site care.

Answer: 1, 2, 3 Explanation: 1. The child will feel full with smaller amounts of food because of the dialysate. 2. The child will be more inclined to eat if there is less stress. 3. Adequate nutrition is important for growth and development, and must be supported if oral intake is inadequate. 4. This intervention is appropriate to prevent infection; it is not a nutritional intervention. 5. This intervention is appropriate to prevent infection; it is not a nutritional intervention.

18) Which should the nurse assess to determine oxygenation during the respiratory assessment for a pediatric client? Select all that apply. 1. Mucous membranes 2. Nail beds 3. Skin 4. Sclerae 5. Corneas

Answer: 1, 2, 3 Explanation: 1. The nurse assesses the mucous membranes to determine oxygenation during the respiratory assessment for a pediatric client. 2. The nurse assesses the nail beds to determine oxygenation during the respiratory assessment for a pediatric client. 3. The nurse assesses the skin to determine oxygenation during the respiratory assessment for a pediatric client. 4. The sclerae are not assessed to determine oxygenation during the respiratory assessment for a pediatric client. 5. The corneas are not assessed to determine oxygenation during the respiratory assessment for a pediatric client.

18) Which actions should the nurse implement when assessing the physical growth for a child who is diagnosed with chronic renal failure? Select all that apply. 1. Asking the child to step on the scale 2. Measuring the child's height 3. Measuring the child's head circumference 4. Using the Denver II with the child 5. Monitoring the child's blood pressure

Answer: 1, 2, 3 Explanation: 1. Weight is a physical growth assessment parameter the nurse uses for a child diagnosed with chronic renal failure. 2. Height is a physical growth assessment parameter the nurse uses for a child diagnosed with chronic renal failure. 3. Head circumference is a physical growth assessment parameter the nurse uses for a child diagnosed with chronic renal failure. 4. The Denver II is a developmental assessment tool. It is not used to assess physical growth. 5. Blood pressure is not a criterion used to measure physical growth.

22) Which injury prevention strategies should the nurse include in the plan of care for a pediatric client who is diagnosed with muscular dystrophy? 1. Develop a home fire evacuation plan. 2. Provide information regarding oxygen safety. 3. Recommend the use of portable generator. 4. Teach safe transfer methods. 5. Perform neurovascular checks every 2 hours.

Answer: 1, 2, 3, 4 Explanation: 1. Helping the family to develop a home fire evacuation plan is an injury prevention strategy the nurse should include in the plan of care. 2. Providing information regarding oxygen safety is an injury prevention strategy the nurse should include in the plan of care. 3. Recommending the use of a portable generator is an injury prevention strategy the nurse should include in the plan of care. 4. Teaching safe transfer methods is an injury prevention strategy the nurse should include in the plan of care. 5. Performing neurovascular checks is appropriate to include in the plan of care for a client who is receiving traction or casting, not for a client diagnosed with muscular dystrophy. Page Ref: 1542

21) Which pediatric client diagnoses would cause the nurse to include information related to short stature? Select all that apply. 1. Hypothyroidism 2. Turner syndrome 3. Chronic renal failure 4. Cushing syndrome 5. Diabetes mellitus

Answer: 1, 2, 3, 4 Explanation: 1. Hypothyroidism is a pediatric client diagnosis that would cause the nurse to include information related to short stature. 2. Turner syndrome is a pediatric client diagnosis that would cause the nurse to include information related to short stature. 3. Chronic renal failure is a pediatric client diagnosis that would cause the nurse to include information related to short stature. 4. Cushing syndrome is a pediatric client diagnosis that would cause the nurse to include information related to short stature. 5. Diabetes mellitus is not a pediatric client diagnosis that would cause the nurse to include information related to short stature.

12) Which risks of undescended testes should the nurse include in the teaching session for the parents of a newborn diagnosed with this condition? Select all that apply. 1. Sperm production will be affected after puberty. 2. Abdominal testes are subject to injury. 3. Abdominal testes have a higher risk of developing cancer. 4. Hormonal production will be affected. 5. The testes are at greater risk of torsion.

Answer: 1, 2, 3, 5 Explanation: 1. Sperm production by abdominal testes is affected by the heat of the body. 2. Positioning of the testes in the scrotum reduces the risk of injury. 3. Statistics have shown this statement is correct. 4. Production of hormones is not affected by the location of the testes. 5. Abdominal testes have a higher risk of twisting on its blood supply.

24) Which diagnostic tests should the nurse include in the plan of care for a pediatric client who is at risk for short stature? Select all that apply. 1. Thyroid function studies 2. Adrenocorticotropic hormone (ACTH) and cortisol levels 3. Complete blood count 4. Blood culture 5. Urine creatinine

Answer: 1, 2, 3, 5 Explanation: 1. Thyroid function tests are often included in the plan of care for a pediatric client at risk for short stature. 2. ACTH and cortisol levels are often included in the plan of care for a pediatric client at risk for short stature. 3. A complete blood count often included in the plan of care for a pediatric client at risk for short stature. 4. A blood culture is not included in the plan of care for a pediatric client at risk for short stature. 5. A urine creatinine is often included in the plan of care for a pediatric client at risk for short stature.

23) Which factors in the maternal medical history should cause the nurse concern regarding the development of cleft lip or cleft palate during pregnancy? Select all that apply. 1. Cigarette smoking 2. Alcohol use 3. Excessive folate intake 4. Glucocorticoid use 5. Anticoagulant use

Answer: 1, 2, 4 Explanation: 1. Cigarette smoking during pregnancy is a risk factor for cleft lip and cleft palate. 2. Alcohol use during pregnancy is a risk factor for cleft lip and cleft palate. 3. Excessive folate intake is not a risk factor for cleft lip and cleft palate. A folate deficiency is often the cause for these disorders. 4. Glucocorticoid use is a risk factor for cleft lip and cleft palate. 5. Anticoagulant use is not a risk factor for cleft lip and cleft palate.

14) Which interventions should the nurse include in the plan of care for an adolescent client who is on complete bed rest after spinal fusion surgery secondary to scoliosis to prevent complications associated with immobility? Select all that apply. 1. Encouraging use of the spirometer every 2 hours while the child is awake 2. Log-rolling the client every 2 hours while awake 3. Increasing intake of milk to maintain bone calcium 4. Increasing fruit and grains in the diet 5. Limiting fluid intake to reduce the need to void

Answer: 1, 2, 4 Explanation: 1. Respiratory complications are a common complication of immobility. 2. Turning the client frequently will reduce pressure on bony prominences. 3. Calcium will be pulled from the bones due to immobility. Adding additional calcium in the form of milk will increase the risk of kidney stones. 4. Fruit and grains will provide extra fiber to reduce the risk of complication. 5. Fluid intake should be increased to "flush" the kidneys.

17) Which should the nurse include in the neurovascular assessment for an infant following casting of the leg for talipes equinovarus? 1. Warmth 2. Capillary refill 3. Pedal pulse 4. Sensation 5. Movement of the toes

Answer: 1, 2, 4, 5 Explanation: 1. The temperature of the foot of the casted leg should be compared to the temperature of the other foot. 2. This indicates blood return to the tissues and is an important finding. 3. The pedal pulse cannot be reached in the casted foot. 4. Nerve function is evaluated by touching the toes and noting the child's response. 5. The child is encouraged to wiggle the toes. If the client is an infant, tickling will cause the child to respond with movement.

19) Which nursing actions are appropriate when providing care to a pediatric client who has sustained a smoke-inhalation injury? Select all that apply. 1. Assessing for respiratory distress 2. Auscultating the lungs for wheezing 3. Prescribing oxygen for low saturations 4. Administering prescribed prophylactic antibiotic therapy 5. Providing support to the family

Answer: 1, 2, 5 Explanation: 1. A pediatric client who sustained a smoke-inhalation injury is at risk for respiratory distress; therefore, it is appropriate for the nurse to assess this patient for clinical manifestations associated with the phenomenon. 2. Crackles and wheezing are both complications associated with a smoke-inhalation injury. This nursing action is appropriate. 3. It is outside the scope of nursing practice to prescribe oxygen therapy for a pediatric client. The nurse would, however, administer prescribed oxygen for this client. 4. Prophylactic antibiotic therapy is not included in the treatment plan for a pediatric client who sustained a smoke-inhalation injury. 5. The nurse should provide support to the family of a pediatric client who sustained a smoke-inhalation injury.

19) Which pediatric client diagnoses necessitate close monitoring for respiratory acidosis? Select all that apply. 1. Aspiration 2. Epiglottitis 3. Sepsis 4. Meningitis 5. Cystic fibrosis

Answer: 1, 2, 5 Explanation: 1. Aspiration places the pediatric client at risk for respiratory acidosis. 2. Epiglottitis places the pediatric client at risk for respiratory acidosis. 3. Sepsis places the pediatric client at risk for respiratory alkalosis. 4. Meningitis places the pediatric client at risk for respiratory alkalosis. 5. Cystic fibrosis places the pediatric client at risk for respiratory acidosis.

20) Which are appropriate interventions for the nurse to include in the plan of care for a child who is receiving traction? Select all that apply. 1. Monitoring breath sounds 2. Assessing neurovascular status every 2 hours 3. Repositioning every 2 to 3 hours 4. Using moleskin to protect the skin from rough edges 5. Encouraging the parents cuddle with their child

Answer: 1, 2, 5 Explanation: 1. Children who are receiving traction are at risk for atelectasis and pneumonia; therefore, the nurse should monitor breath sounds frequently. 2. Child who are receiving traction are at risk for circulatory compromise; therefore, the nurse should perform neurovascular checks every 2 hours. 3. Repositioning every 2 to 3 hours is more appropriate for a child who is casted. 4. Using moleskin to protect the skin from rough edges is more appropriate for a child who is casted. 5. Children who are receiving traction should be allowed up to 1 hour per day without the traction device, at which time the child can eat and cuddle with parents. Page Ref: 1523

15) Which changes should the school nurse implement to decrease the risk for the development of type 2 diabetes mellitus for a population who is identified as being at risk? Select all that apply. 1. Increase the amount of daily physical activity. 2. Meet with all parents and explain the risk that is associated with obesity. 3. Test each child's urine monthly. 4. Teach the parents to avoid administering aspirin to their children. 5. Work with the cafeteria to decrease the amount of fat in the foods served.

Answer: 1, 2, 5 Explanation: 1. Increased physical activity will decrease a child's risk of developing type 2 diabetes. 2. Obese children have an increased risk of type 2 diabetes. Working with the parents, the nurse can reduce the obesity in the school. 3. Testing urine will not decrease the risk of developing type 2 diabetes, although it may lead to earlier diagnosis of the disease. 4. Aspirin administration is not related to type 2 diabetes. 5. A diet high in fat is associated with type 2 diabetes.

12) The nurse is teaching the caregiver of a child who is newly diagnosed with type 1 diabetes mellitus how to minimize pain with insulin injections. Which interventions should the nurse include in the teaching session? Select all that apply. 1. Do not reuse needles. 2. Remove all bubbles from the syringe before injecting. 3. Have the child flex the muscle during injection. 4. Inject insulin when it is cold. 5. Do not change the direction of the needle during insertion or withdrawal.

Answer: 1, 2, 5 Explanation: 1. Reusing needles leads to more pain on injection. 2. Removing bubbles from the syringe minimizes pain. 3. Flexing or tensing muscles during injection causes more discomfort. 4. Insulin should be injected when it is at room temperature to minimize pain. 5. Keeping the direction of the syringe constant will minimize pain.

16) Which assessment data obtained by the nurse during the health history portion of the assessment process support the current diagnosis of Duchenne muscular dystrophy (MD) for an 18-month-old child? 1. Infant was postmature by almost 2 weeks. 2. The child seems very muscular. 3. The child walked early and without support at 10 months. 4. The child's older sister developed scoliosis in the fourth grade.

Answer: 2 Explanation: 1. Postmaturity is not related to Duchenne MD. 2. Duchenne MD is also called pseudohypertrophic due to the enlarged appearance of the muscle. The pathophysiology is infiltration of the muscle fibers with fatty tissue. 3. This finding is not indicative of Duchenne MD. 4. The older sister's scoliosis is not related to MD. Duchenne MD is sex-linked recessive and affects only boys.

20) Which statements should the nurse include in a presentation related to the general function of the gastrointestinal (GI) system for parents of pediatric clients? Select all that apply. 1. "The GI tract is responsible for the ingestion and absorption of food." 2. "Newborns have smaller stomachs but increased peristalsis." 3. "All children require smaller, more frequent feedings." 4. "Infants lack certain digestive enzymes which increases the risk for regurgitation." 5. "By the second year of life a child is able to accommodate three meals each day."

Answer: 1, 2, 5 Explanation: 1. This statement is correct. The GI system is responsible for the ingestion and absorption of food. 2. This statement is correct. Newborns have smaller stomachs but an increased rate of peristalsis. 3. This statement is false. All children do not require smaller, more frequent feedings. This statement is true for newborns and infants. 4. This statement is false. While infants do lack certain digestive enzymes, this does not increase regurgitation but causes abdominal distention due to gas. 5. This statement is true. By the second year of life children are able to accommodate three

17) Which assessment questions should the nurse include in the psychosocial assessment to determine the effects of chronic renal failure treatments on the growth and development of a school-age child? Select all that apply. 1. "How does it make you feel to have to follow a special diet?" 2. "Do you take your medications every day?" 3. "How does it make you feel to undergo dialysis treatments?" 4. "Do you attend school each day?" 5. "How does it make you feel when your parents come home late from work?"

Answer: 1, 3 Explanation: 1. School-age children are often embarrassed about being seen as different from peers. It is appropriate for the nurse to ask the child how it feels to have to follow a special diet. 2. While it is important to assess medication use, this question is not appropriate for the psychosocial portion of the assessment. 3. School-age children are often embarrassed about being seen as different from peers. It is appropriate for the nurse to ask the child how it feels to have to undergo dialysis treatments. 4. While it is important to determine if the child attends school every day, this question is not appropriate for the psychosocial portion of the assessment. 5. This question will not help the nurse to determine the effects of the treatments for chronic renal failure on the child's growth and development.

23) Which functions of the adrenal androgens should the nurse include in a teaching session for a pediatric client diagnosed with alterations in adrenal function? Select all that apply. 1. Stimulates bone development 2. Increases sodium ion reabsorption 3. Stimulates secondary sexual characteristics 4. Increases potassium excretion by the kidneys 5. Activates the sympathetic nervous system

Answer: 1, 3 Explanation: 1. This is a function of androgens. 2. This is a function of aldosterone, not androgens. 3. This is a function of androgens. 4. This is a function of aldosterone, not androgens. 5. This is a function of epinephrine, not androgens.

22) Which topics should the nurse include in discharge instructions related to enhanced safety for a pediatric client who experienced an abdominal injury after a biking accident? Select all that apply. 1. Use of hand signals 2. Age-appropriate use of child safety seats 3. Age-appropriate bicycles 4. Use of a helmet 5. Avoid assigning blame

Answer: 1, 3, 4 Explanation: 1. Information related to appropriate hand signals when riding a bicycle is an injury prevention strategy that the nurse should include in the teaching session. 2. The use of an age-appropriate child safety seat is not an appropriate discharge instruction for a child who experienced an abdominal injury after a biking accident. 3. Information related to an age-appropriate bicycle is an injury prevention strategy that the nurse should include in the teaching session. 4. Information related to the use of a helmet is an injury prevention strategy that the nurse should include in the teaching session. 5. While the nurse should avoid assigning blame when providing care for a child who experienced an abdominal injury as a result of a biking accident, this is not an appropriate injury prevention topic to include in the discharge teaching session.

21) Which are appropriate interventions for the nurse to include in the plan of care for a child who is casted? Select all that apply. 1. Monitoring breath sounds 2. Assessing neurovascular status every 4 hours 3. Repositioning every 2 to 3 hours 4. Using moleskin to protect the skin from rough edges 5. Encouraging the parents cuddle with their child

Answer: 1, 3, 4, 5 Explanation: 1. Children who are casted are at risk for atelectasis and pneumonia; therefore, the nurse should monitor breath sounds frequently. 2. Child who are casted are at risk for circulatory compromise; therefore, the nurse should perform neurovascular checks every 2, not 4, hours. 3. Repositioning every 2 to 3 hours is appropriate for a child who is casted. 4. Using moleskin to protect the skin from rough edges is appropriate for a child who is casted. 5. Children who are casted should be allowed to cuddle with parents to promote developmentally appropriate care.

11) A neonate is born with a bilateral cleft lip that was not detected during the pregnancy. The parents are distressed about the appearance of their infant. Which nursing actions are appropriate to assist the parents to bond with their newborn? Select all that apply. 1. Calling the newborn by the chosen name 2. Keeping the newborn's lower face covered with the blanket 3. Smiling and talking to the newborn in the parents' presence 4. Showing the parents before and after pictures of other children with cleft lips 5. Discussing positive features of the baby

Answer: 1, 3, 4, 5 Explanation: 1. This behavior humanizes the child to the parents and is appropriate. 2. This indicates that the infant's appearance is distressing. Although the nurse would want to shield the child from a visitor's stare, the nurse would not want to hide the child from her own family. 3. This indicates acceptance of the infant by the nurse. 4. It is usually not appropriate to show before and after pictures as you cannot predict the success of the surgery on this child. But in the case of cleft lip, the improvement will be significant and it is considered acceptable to show before and after pictures. 5. Statements like, "Your baby is the sweetest thing—she never cries," can help the parents recognize positive features about their baby.

2) The nurse is providing information to an adolescent newly diagnosed with diabetes. Which clinical manifestations of diabetic ketoacidosis (DKA) should the nurse include in the teaching session? Select all that apply. 1. Change in mental status 2. Tachycardia 3. Fruity breath odor 4. Rapid, shallow respirations 5. Abdominal pain

Answer: 1, 3, 5 Explanation: 1. A change in mental state can be associated with DKA. 2. Tachycardia is not a typical symptom of DKA. 3. A fruity breath odor is common when the client is in a state of ketoacidosis. 4. Respirations are rapid, but deep (Kussmaul breathing) in DKA. 5. Abdominal pain is commonly seen with DKA.

19) Which nursing actions are appropriate to assess growth and development for an adolescent client diagnosed with chronic renal failure? Select all that apply. 1. Using the Denver II during a health maintenance visit 2. Educating parents on normal milestones 3. Monitoring for delayed sexual maturation 4. Comparing blood pressure values from previous visit 5. Plotting height and weight measurements

Answer: 1, 3, 5 Explanation: 1. The Denver II is a developmental assessment tool that is appropriate for the nurse to use when assessing growth and development for an adolescent client diagnosed with chronic renal failure. 2. It is appropriate for the nurse to educate the client's parents on normal milestones; however, this is not a nursing assessment. 3. Monitoring for delayed sexual maturation is appropriate when assessing growth and development for an adolescent client diagnosed with chronic renal failure. 4. Blood pressure is not a growth and development parameter. 5. Plotting height and weight measurements is an appropriate nursing action to assess growth and development for an adolescent client diagnosed with chronic renal failure.

11) Which assessment finding would necessitate action by the nurse for a 10-month-old child who is 4 hours postoperative for the placement of a urethral stent? 1. Bloody urine 2. One void since returning from surgery 3. Bladder spasms responding to pharmacologic intervention 4. Double diapering from the previous shift

Answer: 2 Explanation: 1. Bloody urine is expected in the immediate postoperative period. 2. A 10-month-old child will void more often than 1 time in 4 hours. This could indicate the stent is occluded. The surgeon should be notified. 3. This is a normal finding. 4. This is a desired finding and does not need to be reported to the surgeon.

4) The nurse is assessing an infant brought to the clinic because of diarrhea. The infant is alert but has dry mucous membranes. Which additional assessment data indicates to the nurse that the infant is experiencing an early to moderate stage of dehydration? 1. Bradycardia 2. Tachycardia 3. Increased blood pressure 4. Normal fontanels

Answer: 2 Explanation: 1. Bradycardia is not a sign of dehydration. 2. Tachycardia is a sign that indicates moderate dehydration. 3. In dehydration, the blood pressure is hypotensive. 4. Fontanels would be sunken in moderate dehydration.

17) The nurse is providing care to a newborn who is suspected of having Turner syndrome. Which should the nurse assess the newborn for based on the current diagnosis? 1. Club foot (talipes equinovarus) 2. Congenital heart anomalies 3. Hyperbilirubinemia due to liver abnormalities 4. Diaphragmatic hernia

Answer: 2 Explanation: 1. Club foot is not associated with Turner syndrome. 2. Congenital heart anomalies, including coarctation of the aorta, frequently are associated with Turner syndrome. 3. The newborn with Turner syndrome has the normal risk for hyperbilirubinemia. 4. Diaphragmatic hernias are not associated with Turner syndrome.

3) Which urinalysis result should the nurse anticipate for a child who is admitted with acute glomerulonephritis? 1. Bacteriuria and increased specific gravity 2. Hematuria and proteinuria 3. Proteinuria and decreased specific gravity 4. Bacteriuria and hematuria

Answer: 2 Explanation: 1. Glomerulonephritis is an inflammation of the glomeruli of the kidneys. Bacteriuria is not present. But because the urine is concentrated, the specific gravity is increased. 2. Glomerulonephritis is an inflammation of the glomeruli of the kidneys. The clinical manifestation of glomerulonephritis is grossly bloody hematuria with mild to moderate proteinuria, and because the urine is concentrated, the specific gravity is increased. 3. Glomerulonephritis is an inflammation of the glomeruli of the kidneys. Because the urine is concentrated, the specific gravity is increased. 4. Glomerulonephritis is an inflammation of the glomeruli of the kidneys. Bacteriuria is not present.

14) Which sequela should the nurse include in the teaching session for a parent who does not believe in medication for the treatment of the newborn's hypothyroidism? 1. Heart disease 2. Mental retardation 3. Renal failure 4. Thyroid storm

Answer: 2 Explanation: 1. If the hypothyroidism is left untreated, the child will experience bradycardia but will not develop heart disease. 2. Untreated hypothyroidism will lead to mental retardation. 3. Untreated hypothyroidism does not lead to renal failure. 4. Thyroid storm is a complication of hyperthyroidism, not hypothyroidism.

9) Which should the nurse include in a teaching session for the parents of an infant who will be placed in a Pavlik harness for the treatment of congenital developmental dysplasia? 1. Apply lotion or powder to minimize skin irritation. 2. Check at least 2 or 3 times a day for red areas under the straps. 3. Put clothing over the harness for maximum effectiveness of the device. 4. Place a diaper over the harness, preferably using a thin, superabsorbent, disposable diaper.

Answer: 2 Explanation: 1. Lotion or powder can contribute to skin breakdown and should not be used. 2. The skin underneath the straps of the brace should be checked 2 or 3 times a day for red areas, which might indicate skin breakdown. 3. A light layer of clothing should be worn under the brace to assist in preventing skin breakdown, not over the brace. 4. The diaper should be placed under the brace, along with a light layer of clothing.

3) An adolescent client reports recurrent abdominal pain with diarrhea and bloody stools. Which type of inflammatory bowel disease does the nurse suspect based on these data? 1. Necrotizing enterocolitis (NEC) 2. Ulcerative colitis (UC) 3. Crohn disease 4. Appendicitis

Answer: 2 Explanation: 1. NEC is usually seen in premature infants and generally not in an adolescent client. 2. Diarrhea and bloody stools are typical symptoms of UC. 3. The teen with Crohn disease might have abdominal pain and diarrhea, but stools usually do not have blood in them. 4. Appendicitis is not associated with bloody stools and usually not with diarrhea.

4) Which nursing action is appropriate for the parents of a 4-month-old infant who died due to sudden infant death syndrome (SIDS)? 1. Sheltering parents from the grief by not giving them any personal items of the infant, such as footprints 2. Allowing parents to hold, touch, and rock the infant 3. Advising parents that an autopsy is not necessary 4. Interviewing parents to determine the cause of the incident

Answer: 2 Explanation: 1. Parents will want any personal items available. 2. The parents should be allowed to hold, touch, and rock the infant, giving them a chance to say good-bye to their baby. 3. The death of an infant without a known medical condition is an indication for an autopsy. 4. The parents need to know that SIDS is not their fault.

1) The home health nurse is visiting a 3-month-old infant who is diagnosed with congenital hypothyroidism and is prescribed daily levothyroxine. Which should the nurse include in the infant's continued plan of care? 1. Stopping the medication as long as the child continues to grow 2. Preventing hypothermia with appropriate clothing 3. Changing formula because it is contraindicated with prescribed medication 4. Monitoring growth and development without any other prescribed interventions

Answer: 2 Explanation: 1. The medication must be continued for life. 2. The parents should be cautioned to dress the child appropriately to prevent hypothermia. 3. The infant formula is not contraindicated with the prescribed medication. 4. The child will continue to need monitoring and intervention even if growth and development are not affected.

12) Which nursing action is appropriate when providing care to a newborn with a respiratory rate of 102 breaths per minute with lungs that are clear to auscultation? 1. Administering the bath to the neonate in the nursery 2. Transferring to the neonatal intensive care unit for further observation 3. Allowing the neonate to room-in to promote bonding 4. Providing the first feeding in the nursery

Answer: 2 Explanation: 1. The newborn is tachypneic. Bathing will only add to the respiratory distress and should be avoided. 2. This newborn needs to remain under constant observation due to the respiratory rate. 3. The newborn needs to be monitored. 4. With a respiratory rate this high, aspiration is likely so feeding should be avoided.

13) Which assessment finding, after the dialysate is drained during peritoneal dialysis for a child experiencing acute renal failure, would warrant further action by the nurse? 1. The dialysate is clear on return. 2. The volume of drained dialysate is less than the volume infused. 3. The child is restless, wanting to get up and play. 4. The child's vital signs are basically the same as were noted on infusion.

Answer: 2 Explanation: 1. This is a normal finding and does not require reporting. 2. This indicates fluids are being retained and is not desirable. The healthcare provider should be notified. 3. This could indicate the child is feeling better. It is a desired effect and does not require reporting to the healthcare provider. 4. This is an expected finding. No dramatic differences in vital signs should be noted.

5) Which assessment finding would require an immediate nursing action when providing care to an adolescent who is postoperative for spinal fusion surgery? 1. Sleeps when not bothered but arouses easily with stimuli 2. Impaired color, sensitivity, and movement to lower extremities 3. Nausea relieved by antiemetics 4. Pain relieved by analgesics

Answer: 2 Explanation: 1. This is a normal response postanesthesia. 2. When the spinal column is manipulated, there is a risk for impaired color, sensitivity, and movement to lower extremities. 3. Nausea in the postoperative period is not uncommon, but it is not the priority at this time. 4. Pain is a common finding in the postoperative period and should be addressed, but impaired color, sensitivity, and movement of the lower extremities constitute the priority at this time.

20) The nurse is providing care to a newborn female who is born with ambiguous genitalia. The follow-up investigation discovers adrenogenital syndrome (also called congenital adrenal hyperplasia [CAH]). The parents question why the baby's genitalia looks more male than female. Which response by the nurse is accurate? 1. "The disorder caused your baby to be a hermaphrodite with both male and female sex organs." 2. "The changes in the genitalia are due to increased androgens secondary to deficient cortisol." 3. "The excessive cortisol caused the enlargement of the female tissue, creating a male appearance." 4. "Your baby has only one sex chromosome resulting in an XO configuration.

Answer: 2 Explanation: 1. This statement is incorrect. The child's internal organs will be ovaries only. 2. Deficient cortisol causes the amount of adrenocorticotropic hormone (ACTH) to be high, overstimulating the adrenal production of androgens, which causes the pseudomasculinization. 3. The cortisol level is decreased, not increased. 4. XO sex chromosomes describe Turner syndrome, not CAH.

7) Which teaching topic is the priority for the nurse who is teaching the family of an infant diagnosed with osteogenesis imperfecta? 1. Cast care 2. Trunk and extremity support during everyday care 3. Postoperative spinal surgery care 4. Traction care

Answer: 2 Explanation: 1. Traction, casts, and spinal surgery are not routinely done for osteogenesis imperfecta. 2. With osteogenesis imperfecta, nursing care focuses on preventing fractures. Because the bones are fragile, the entire body must be supported when the child is moved. 3. Traction, casts, and spinal surgery are not routinely done for osteogenesis imperfecta. 4. Traction, casts, and spinal surgery are not routinely done for osteogenesis imperfecta.

2) Which is the appropriate nursing intervention when providing care to a child, diagnosed with nephrotic syndrome, who is edematous and on bed rest? 1. Monitor blood pressure every 30 minutes. 2. Reposition every 2 hours. 3. Limit visitors. 4. Encourage fluids.

Answer: 2 Explanation: 1. Vital signs are taken every 4 hours. 2. A child with severe edema, on bed rest, is at risk for altered skin integrity. To prevent skin breakdown, the child should be repositioned every 2 hours. 3. The child needs social interaction, so visitors should not be limited. 4. Fluids need to be monitored; they should not be encouraged.

8) An infant returns to the unit following surgical correction of bilateral congenital clubfeet. The infant has bilateral long-leg casts. The nurse notes that the toes on both feet are edematous, but there is color, sensitivity, and movement to them. Which action by the nurse is the priority? 1. Apply a warm, moist pack to the feet. 2. Elevate the legs on pillows. 3. Encourage movement of the toes. 4. Call the surgical provider to report the edema.

Answer: 2 Explanation: 1. Warm, moist heat will increase swelling and the moisture may cause the cast to disintegrate. 2. The infant's legs should be elevated on a pillow for 24 hours to promote healing and help with venous return. This is the priority action. 3. An infant would not be able to follow directions to move the toes, and in this case it would not be as effective as would elevating the legs on pillows. 4. Some amount of swelling can be expected, so it would not be appropriate to notify the physician, especially if the color, sensitivity, and movement to the toes remained normal.

13) A nasogastric tube to suction is ordered for a neonate diagnosed with a diaphragmatic hernia. Which complication related to gastric drainage is the priority when planning care for this neonate? 1. Weight loss 2. Metabolic alkalosis 3. Dehydration 4. Hyperbilirubinemia

Answer: 2 Explanation: 1. Weight loss and inadequate nutrition are not the priority for this client. 2. When large quantities of gastric juice is removed, acid is lost and metabolic alkalosis follows. 3. The volume would not be sufficient to cause dehydration. 4. Hyperbilirubinemia is unrelated to gastric suction.

12) The father of a school-age child who requires hospital admission for intravenous antibiotics to treat osteomyelitis states, "I don't understand why normal antibiotics can't be used." Which should the nurse include in the response to the father? 1. The antibiotic of choice is not available in oral form. 2. Blood flow to bones is limited, and parenteral administration is necessary to get appropriate blood levels. 3. Because the child is older now, it is harder to get the child to cooperate with oral antibiotics. 4. Because 2 weeks of therapy is necessary, the intravenous route will produce fewer side effects.

Answer: 2 Explanation: 1. Most antibiotics are available in multiple forms. 2. This is accurate information. 3. The older child can understand the reason for antibiotics and cooperate. 4. Both oral and intravenous antibiotics may have side effects.

20) Which pediatric clients would require a nursing assessment for blunt chest trauma? Select all that apply. 1. A preschool-age client who is admitted after a house fire. 2. A toddler-age client who is admitted for injuries sustained in a motor vehicle accident. 3. A school-age client who is admitted for observation after a skateboarding accident. 4. An adolescent client admitted for an asthma exacerbation. 5. An infant admitted to rule out cystic fibrosis.

Answer: 2, 3 Explanation: 1. A preschool-age client admitted after a house fire would require assessment for smoke-inhalation injury not blunt chest trauma. 2. A toddler-age client admitted for injuries sustained in a motor vehicle accident would require assessment to determine blunt chest trauma. 3. A school-age client admitted for observation after a skateboarding accident would require assessment to determine blunt chest trauma. 4. An asthma exacerbation would not necessitate a nursing assessment for blunt chest trauma. 5. An infant admitted to rule out cystic fibrosis would not necessitate a nursing assessment for blunt chest trauma.

18) Which clinical manifestations should the nurse expect when assessing a pediatric client who is diagnosed with congenital hip dysplasia (CHD)? Select all that apply. 1. Limited adduction of the affected hip 2. Asymmetry of thigh fat folds 3. Telescoping of the thigh 4. Muscle weakness 5. Atrophy of the muscles

Answer: 2, 3 Explanation: 1. The nurse would anticipate limited abduction, not adduction, of the affected hip for a child diagnosed with CHD. 2. Asymmetry of the thigh fat folds is a clinical manifestation associated with CHD. 3. Telescoping of the thigh is a clinical manifestation associated with CHD. 4. Muscle weakness is not an expected clinical manifestation associated with CHD. 5. Atrophy of the muscles is not an expected clinical manifestation associated with CHD.

23) Which age-appropriate techniques should the nurse implement in order to encourage a young child to participate in deep breathing exercises? Select all that apply. 1. Showing the child how to use the "blow bottle" 2. Using a pinwheel that the child plays with and asking the child to blow until it turns 3. Asking the child to blow bubbles in a glass of water using a straw 4. Having the child blow scraps of paper across the bedside table with a straw 5. Telling the child that a "shot" will be needed if the child does not follow the nurse's

Answer: 2, 3, 4 Explanation: 1. A blow bottle is appropriate for an older pediatric client, not a young child. 2. Asking the young child to blow on a pinwheel is an age-appropriate intervention to facilitate deep breathing. 3. Asking the child to blow bubbles into a glass of water is an age-appropriate intervention to facilitate deep breathing. 4. Having the child blow scraps of paper across the bedside table with a straw is an age-appropriate intervention to facilitate deep breathing. 5. Telling the child that an injection will be administered if the nurse's directions are not followed is not therapeutic nor age appropriate.

14) Which statements, made by the adolescent following dietary teaching for Crohn disease, indicate correct understanding of the content presented by the nurse? Select all that apply. 1. "I can promote solid stools by increasing fiber in my diet." 2. "Small, frequent meals are preferred over three meals a day." 3. "I should identify foods that cause distress and eliminate them from my diet." 4. "High-calorie dietary supplement shakes can help me to meet my nutritional requirements." 5. "Socialization during my meal times is important even if my parents do not agree with my food choices."

Answer: 2, 3, 4 Explanation: 1. Fiber should be decreased, not increased, as diarrhea is one of the symptoms of Crohn disease. 2. This is correct information. 3. This is individualizing the diet and is appropriate. 4. This addition provides an easy way to meet the nutritional needs. 5. Stress should be avoided at mealtimes.

17) Which data collected during the respiratory assessment would indicate the pediatric client is compromised? Select all that apply. 1. Lung sounds clear to auscultation 2. Stridor 3. Substernal retractions 4. Nasal flaring 5. Strong cry

Answer: 2, 3, 4 Explanation: 1. Lung sounds that are clear to auscultation do not indicate respiratory compromise. 2. Stridor is an adventitious breath sound that may indicate respiratory compromise. 3. Substernal retractions may indicate respiratory compromise. 4. Nasal flaring may indicate respiratory compromise. 5. A weak, not strong, cry may indicate respiratory compromise.

21) Which nursing assessment data would indicate that a pediatric client sustained a large pulmonary contusion in a motor vehicle crash? Select all that apply. 1. Eupnea 2. Dyspnea 3. Hemoptysis 4. Fever 5. Crackles

Answer: 2, 3, 4, 5 Explanation: 1. Eupnea, or a normal respiratory rate, is not assessment data the nurse expects for a pediatric client who sustained a large pulmonary contusion in a motor vehicle crash. 2. Dyspnea is a clinical manifestation associated with respiratory distress, which can occur for the pediatric client who sustained a large pulmonary contusion in a motor vehicle crash. 3. Hemoptysis is a clinical manifestation associated with a large pulmonary contusion. 4. Fever is a clinical manifestation associated with a large pulmonary contusion. 5. Crackles are a clinical manifestation associated with a large pulmonary contusion.

10) Which complications should the nurse monitor for when providing care to a child who is having hemodialysis for the treatment of kidney failure? Select all that apply. 1. Migraines 2. Hypotension 3. Infections 4. Fluid overload 5. Shock

Answer: 2, 3, 5 Explanation: 1. Migraines are not a clinical manifestation associated with hemodialysis. 2. Rapid changes in fluid and electrolyte balance during hemodialysis can lead to hypotension. 3. Infection is another complication that may occur during hemodialysis. 4. Fluid overload is not a clinical manifestation associated with hemodialysis. 5. Rapid changes in fluid and electrolyte balance during hemodialysis can lead to shock.

A 9-month-old infant is hospitalized with vomiting and diarrhea. The mother questions why her child needed hospitalization since her school-age nephew had the same symptoms and was treated at home. Which should the nurse include in the explanation to the infant's mother? Select all that apply. 1. Infants have a lower proportion of their body weight as water. 2. The percentage of extracellular fluid is higher in the infant than the school-age child. 3. School-age children have a larger body surface area. 4. The school-age child's kidneys are more mature and better able to conserve water. 5. The metabolic rate of the school-age child is higher.

Answer: 2, 4 Explanation: 1. Infants have a higher percentage of body weight as water. 2. This statement is accurate. 3. Body surface area (BSA) is an assessment of skin surface. BSA compares the height and weight of the child and is greatest in infancy. 4. This statement is accurate. 5. Infants have a higher metabolic rate than a school-age child.

22) Which functions of the adrenal hormone aldosterone should the nurse include in a teaching session for a pediatric client diagnosed with alterations in adrenal function? Select all that apply. 1. Stimulates bone development 2. Increases sodium ion reabsorption 3. Stimulates secondary sexual characteristics 4. Increases potassium excretion by the kidneys 5. Activates the sympathetic nervous system

Answer: 2, 4 Explanation: 1. This is a function of androgens, not aldosterone. 2. This is a function of aldosterone. 3. This is a function of androgens, not aldosterone. 4. This is a function of aldosterone. 5. This is a function of epinephrine, not aldosterone. Page Ref: 1399

2) Which clinical manifestations should the nurse monitor for when conducting a scoliosis screening for a school-age child? Select all that apply. 1. Lordosis 2. Prominent scapula 3. Pain 4. A one-sided rib hump 5. Uneven shoulders and hips

Answer: 2, 4, 5 Explanation: 1. Lordosis is not present with scoliosis. 2. The classic signs of scoliosis include uneven shoulders and hips, a one-sided rib hump, and prominent scapula. 3. Pain generally is not present with scoliosis unless it is severe. 4. The classic signs of scoliosis include uneven shoulders and hips, a one-sided rib hump, and prominent scapula. 5. The classic signs of scoliosis include uneven shoulders and hips, a one-sided rib hump, and prominent scapula.

14) Which instructions should be provided to the parents of a 4-year-old girl who has experienced chronic urinary tract infections (UTIs) in the last 2 years? Select all that apply. 1. Wear only nylon underwear for better air flow. 2. Teach the child to wipe from front to back. 3. Encourage the child to take long baths by allowing the child bubbles and toys in the tub. 4. Encourage the child to drink additional fluids throughout the day. 5. Plan potty breaks every 2 hours throughout the day.

Answer: 2, 4, 5 Explanation: 1. The child should wear cotton underwear. 2. This prevents bacteria from the rectum from being introduced into the urethra. 3. Bubble baths should be avoided. 4. Extra fluids will "wash" bacteria out of the bladder. 5. Children get so involved in playing that they often hold their urine. Voiding every 2 hours will reduce the time for bacteria to grow in the bladder.

11) Which positions are appropriate for the nurse to include in a plan of care for the infant who is diagnosed with acute respiratory distress? Select all that apply. 1. Upright 2. Semi-Fowler position 3. Prone position 4. With the infant's head hyperextended 5. With the infant's head in a sniffing position

Answer: 2, 5 Explanation: 1. An infant cannot be placed in an upright position. 2. The semi-Fowler position elevates the head of bed. This allows better movement of the diaphragm. 3. Prone positioning will not promote respirations. 4. The head should not be hyperextended as that position does not open the airway in an infant. 5. A sniffing position straightens and shortens the airway and is the position that is best.

19) The nurse is providing care to a newborn client who presents in the pediatric clinic for a 2-week health maintenance visit. The parents of the newborn are concerned, as their baby has "gas all the time." Which responses from the nurse are appropriate? Select all that apply. 1. "Your baby has a relaxed lower esophageal sphincter, which is causing the gas." 2. "Your baby lacks the enzyme amylase, which is causing the gas." 3. "Your baby lacks the enzyme insulin, which is causing the gas." 4. "Your baby has an immature liver, which is causing the gas." 5. "Your baby lacks an enzyme that helps to digest fats, which is causing the gas."

Answer: 2, 5 Explanation: 1. Newborns and infants do have a relaxed lower esophageal sphincter; however, this is not responsible for gas but for frequent regurgitation of small amounts of oral feedings. 2. Newborns and infants lack several enzymes that assist with the digestive process. One of these enzymes is amylase, which assists with carbohydrate digestion. The lack of this enzyme causes abdominal distention due to gas. 3. Insulin is not an enzyme and is not lacking in the newborn. 4. While newborns and infants do have immature livers, that is not what is causing the gas. 5. Lipase is a digestive enzyme that assists in fat digestion. Infants and newborns do lack this enzyme, which would cause abdominal distention due to gas.

5) Which laboratory tests should the nurse prepare to draw when admitting a pediatric client with possible obstructive uropathy? Select all that apply. 1. Platelet count 2. Blood urea nitrogen (BUN) 3. Partial thromboplastin time (PTT) 4. Blood culture 5. Creatinine

Answer: 2, 5 Explanation: 1. Platelet count is drawn when a bleeding disorder is suspected. 2. BUN is a serum laboratory test for kidney function. Obstructive uropathy is a structural or functional abnormality of the urinary system that interferes with urine flow and results in urine backflow into the kidneys; therefore, the BUN will be elevated. 3. PTT is drawn when a bleeding disorder is suspected. 4. A blood culture is done when an infectious process is suspected. 5. Creatinine is a serum laboratory test for kidney function. Obstructive uropathy is a structural or functional abnormality of the urinary system that interferes with urine flow and results in urine backflow into the kidneys; therefore, the creatinine will be elevated.

15) Which is the priority nursing intervention when caring for a neonate who is born with bladder exstrophy? 1. Measuring intake and output 2. Inserting a Foley catheter 3. Covering the defect with sterile plastic wrap 4. Palpating the bladder mass to ensure urine is expelled

Answer: 3 Explanation: 1. Because the bladder constantly drains onto the skin of the abdomen, measuring output is not possible. 2. The bladder is open to the abdomen. A Foley catheter cannot be inserted. 3. This reduces the contamination of the bladder, which should be sterile. 4. The bladder is very sensitive and palpation would cause unnecessary pain.

2) Which assessment data would cause the nurse to suspect that a 3-year-old child has Hirschsprung disease? 1. Clay-colored stools and dark urine 2. History of early passage of meconium in the newborn period 3. History of chronic, progressive constipation and failure to gain weight 4. Continual bouts of foul-smelling diarrhea

Answer: 3 Explanation: 1. Clay-colored stools and dark urine are not associated with Hirschsprung disease. 2. The infant with Hirschsprung disease often has delayed meconium stools. 3. These are symptoms of Hirschsprung disease in an older infant or child. 4. Diarrhea is not typical; obstruction is more likely.

6) Which clinical manifestations should the nurse anticipate upon assessment for a preschool-age child with a urinary tract infection (UTI)? 1. Headache, hematuria, and vertigo 2. Foul-smelling urine, elevated blood pressure (BP), and hematuria 3. Urgency, dysuria, and fever 4. Severe flank pain, nausea, and headache

Answer: 3 Explanation: 1. Hematuria might be present, but there will be no complaints of headache or vertigo. 2. While foul-smelling urine and hematuria can be present, there is no elevated BP, headache, or vertigo. 3. Clinical manifestations of UTI in a preschool-age child include fever, urgency, and dysuria. 4. There could be flank pain, although the preschooler might be unable to describe it. There will be no complaints of headache.

8) The nurse is caring for a child just admitted with diabetic ketoacidosis (DKA). Which healthcare provider prescription should the nurse question? 1. Neurologic checks hourly 2. Insert urinary catheter and measure output hourly 3. NPH insulin IV at 0.1 unit/kg per hour 4. Stat serum electrolytes

Answer: 3 Explanation: 1. Hourly neurologic checks are an appropriate order. 2. Urinary catheter and hourly outputs are appropriate. 3. NPH insulin is never administered IV. A short-acting insulin needs to be ordered. 4. Stat electrolytes are an appropriate order. Page Ref: 1423

5) Which clinical manifestations should the nurse anticipate when providing care to an adolescent client who presents with untreated Graves disease? 1. Hyperglycemia, ketonuria, and glucosuria 2. Weight gain, hirsutism, and muscle weakness 3. Tachycardia, fatigue, and heat intolerance 4. Dehydration, metabolic acidosis, and hypertension

Answer: 3 Explanation: 1. Hyperglycemia, ketonuria, and glucosuria are signs of diabetes. 2. Weight gain, hirsutism, and muscle weakness are seen in clients with Cushing disease. 3. Clinical manifestations of Graves disease are tachycardia, fatigue, and heat intolerance, seen with hyperthyroidism. 4. Dehydration, metabolic acidosis, and hypertension are signs of congenital adrenal hyperplasia.

1) A 4-year-old child is admitted to the hospital secondary to dehydration. Laboratory tests indicate a high hemoglobin and hematocrit, and the serum sodium is below normal levels. Which condition does the nurse suspect based on the current data? 1. Hypernatremia 2. Metabolic acidosis 3. Hypotonic dehydration 4. Isotonic dehydration

Answer: 3 Explanation: 1. Hypernatremia is a condition where the body fluids are too concentrated and there is an excess of sodium. 2. Metabolic acidosis refers to a condition where the pH of the blood is acidic. 3. This occurs when fluid loss is characterized by a proportionately greater loss of sodium than water. Serum sodium is below normal levels. Hemoglobin and hematocrit will be high due to the loss of serum water. 4. This occurs when fluid loss is not balanced by intake, and the losses of water and sodium are in proportion.

1) Which finding, noted during the newborn admission assessment, would lead the nurse to suspect unilateral congenital hip dysplasia? 1. Lordosis 2. Trendelenburg sign 3. Asymmetry of the gluteal and thigh fat folds 4. Telescoping of the affected limb

Answer: 3 Explanation: 1. Lordosis does not occur with hip dysplasia. 2. Trendelenburg sign and telescoping of the affected limb are signs that present in an older child with congenital hip dysplasia. 3. A sign of congenital hip dysplasia in the infant would be asymmetry of the gluteal and thigh fat folds. 4. Trendelenburg sign and telescoping of the affected limb are signs that present in an older child with congenital hip dysplasia.

8) Which parental statement at the end of a teaching session by the nurse indicates correct understanding of colostomy stoma care for the infant client? 1. "We will change the colostomy bag with each wet diaper." 2. "We will expect a moderate amount of bleeding after cleansing the area around the stoma." 3. "We will watch for skin irritation around the stoma." 4. "We will use adhesive enhancers when we change the bag."

Answer: 3 Explanation: 1. Physical or chemical skin irritation can occur if the appliance is changed too frequently, or with each wet diaper. 2. Bleeding is usually attributable to excessive cleaning. 3. Skin irritation around the stoma should be assessed; it could indicate leakage. 4. Adhesive enhancers should be avoided on the skin of infants. Their skin layers are thin, and removal of the appliance can strip off the skin.

15) Which parental action, observed during a home care visit for an infant diagnosed with gastroesophageal reflux, requires intervention by the nurse? 1. The infant's formula has rice cereal added. 2. The mother holds the infant in a high Fowler position while feeding. 3. After feeding, the infant is placed in a car seat. 4. The mother draws up the ranitidine (Zantac) in a syringe for oral administration.

Answer: 3 Explanation: 1. Rice cereal thickens the formula and helps prevent regurgitation. This is appropriate. 2. This position will help prevent regurgitation and is appropriate. 3. Infant seats are not recommended, as they put pressure on the abdomen and may contribute to regurgitation. 4. Since dosing is small, it is appropriate to use a syringe for accurate measurement.

4) Which is the priority nursing diagnosis for nurse to use when planning care for a school-age child who must wear a brace for correction of scoliosis? 1. Impaired Gas Exchange, Risk for 2. Altered Growth and Development, Risk for 3. Impaired Skin Integrity, Risk for 4. Impaired Mobility, Risk for

Answer: 3 Explanation: 1. Risk for impaired gas exchange is a late effect of scoliosis and would not be the priority. If the client is compliant with wearing the brace, the risk should be minimized. 2. The diagnosis of Altered Growth and Development would not be the priority and should be corrected by the wearing of the brace. 3. The skin should be monitored for breakdown in any area where the brace might rub against the skin; therefore, Risk for Impaired Skin Integrity is the priority nursing diagnosis. 4. The diagnosis of Impaired Mobility would not be the priority and should be corrected if the client is compliant with wearing the brace.

9) An adolescent presents in the emergency department (ED) with confusion. The healthcare provider suspects diabetic ketoacidosis (DKA). A stat serum glucose is done, and the result is 7l5 mg/dL. Which clinical manifestations does the nurse anticipate upon assessment for this client? 1. Tachycardia, dehydration, and abdominal pain 2. Sweating, photophobia, and tremors 3. Dry mucous membranes, blurred vision, and weakness 4. Dry skin, shallow rapid breathing, and dehydration

Answer: 3 Explanation: 1. Tachycardia is seen in hypoglycemia. 2. Sweating, photophobia, and tremors are indicative of hypoglycemia. 3. Dry mucous membranes, blurred vision, and weakness are seen with hyperglycemia. 4. Dry skin and dehydration are signs of hyperglycemia, but shallow breathing is a sign of hypoglycemia. Page Ref: 1422

10) Which should the nurse include in a teaching session for the mother of a 3-year-old client who is concerned about her child choking? 1. Show the mother how to do cardiac compressions and rescue breathing. 2. Recommend the mother perform back blows and chest thrusts. 3. Teach the mother how to perform abdominal thrusts. 4. Tell the mother to do nothing until the child loses consciousness.

Answer: 3 Explanation: 1. The method of cardiac compressions and rescue breathing is not the first thing that the mother needs to know. 2. This is the treatment for a choking infant, not a child. 3. Giving abdominal thrusts is the correct intervention for a choking child. 4. The mother should respond to the choking child before the child loses consciousness.

13) Which assessment finding for a toddler-age child in balanced Bryant traction for a fractured right femur would require immediate action by the nurse? 1. The child keeps trying to turn and lie on his belly. 2. The ropes are unequal in length. 3. The child's buttocks are resting on the bed. 4. The Ace bandage wrapping the legs is wrinkled.

Answer: 3 Explanation: 1. This child needs a jacket restraint to maintain appropriate positioning if someone cannot stay with him. It does not require notifying the surgeon. 2. In balanced traction, the ropes and pulleys determine the traction and the length of the rope is unimportant. 3. In order to provide adequate counter-traction, the buttocks should be slightly elevated off the bed. The surgeon should be notified. 4. This is not a significant finding.

17) A toddler is admitted to the surgical unit for a planned closure of a temporary colostomy. Which medical prescription should the nurse question? 1. Clear liquids today. NPO tomorrow 2. Type and cross-match for 1 unit of packed red blood cells. 3. Rectal temperatures every 4 hours 4. Start an intravenous line with D5NS at 20 mL per hour.

Answer: 3 Explanation: 1. This is appropriate in anticipation of surgery. 2. Although not always required during surgery, this would not be inappropriate planning for the surgical procedure. 3. Rectal temperatures are avoided due to the fragile state of the rectum. 4. An IV is appropriate for surgical access.

9) A child with croup has an increased PCO2, a decreased pH, and a normal HCO3 blood gas value. Which does the nurse report to the healthcare provider based on these data? 1. Uncompensated metabolic alkalosis 2. Uncompensated metabolic acidosis 3. Uncompensated respiratory acidosis 4. Uncompensated respiratory alkalosis

Answer: 3 Explanation: 1. Uncompensated metabolic alkalosis has an increased pH, normal PCO2, and increased HCO3. 2. Uncompensated metabolic acidosis has a decreased pH, normal PCO2, and normal HCO3. 3. If the pH is decreased and the PCO2 is increased with a normal HCO3, it is uncompensated respiratory acidosis. Also, croup can be a disease process that causes respiratory acidosis. 4. Uncompensated respiratory alkalosis has an increased pH, decreased PCO2, and normal HCO3.

20) Which pediatric client diagnoses necessitate close monitoring for respiratory alkalosis? Select all that apply. 1. Aspiration 2. Epiglottitis 3. Sepsis 4. Meningitis 5. Cystic fibrosis

Answer: 3, 4 Explanation: 1. Aspiration places the pediatric client at risk for respiratory acidosis. 2. Epiglottitis places the pediatric client at risk for respiratory acidosis. 3. Sepsis places the pediatric client at risk for respiratory alkalosis. 4. Meningitis places the pediatric client at risk for respiratory alkalosis. 5. Cystic fibrosis places the pediatric client at risk for respiratory acidosis.

11) Which clinical data noted by the nurse during the shift assessment indicate the pediatric client may be experiencing compartment syndrome? Select all that apply. 1. Pink, warm extremity 2. Dorsalis pedis pulse present 3. Prolonged capillary refill time 4. Pain not relieved by pain medication 5. Paresthesia of the leg

Answer: 3, 4, 5 Explanation: 1. Pink, warm extremity is a normal finding post fracture reduction. 2. A present dorsalis pedis pulse would be a normal finding post fracture reduction. 3. A prolonged capillary refill time is a sign of compartment syndrome. 4. A prolonged capillary refill time with loss of paresthesia and pain not relieved by medication are signs of compartment syndrome. 5. Paresthesia is tingling and numbness of the affected extremity and is a sign of compartment syndrome.

25) Which assessment data for a pediatric client supports the diagnosis of familial or idiopathic central diabetes insipidus (DI)? Select all that apply. 1. Polyuria 2. Polydipsia 3. Nocturia 4. Enuresis 5. Constipation

Answer: 3, 4, 5 Explanation: 1. Polyuria is not a clinical manifestation associated with familial or idiopathic central DI. 2. Polydipsia is not a clinical manifestation associated with familial or idiopathic central DI. 3. Nocturia is a clinical manifestation associated with familial or idiopathic central DI. 4. Enuresis a clinical manifestation associated with familial or idiopathic central DI. 5. Constipation a clinical manifestation associated with familial or idiopathic central DI.

18) Which gastrointestinal defects, often diagnosed shortly after birth, should the nurse include in the assessment process of all newborns? Select all that apply. 1. Pyloric stenosis 2. Biliary atresia 3. Hirschsprung disease 4. Umbilical hernia 5. Diaphragmatic hernia

Answer: 3, 5 Explanation: 1. Pyloric stenosis is not diagnosed in the newborn nursery, but in the 2- to 4-week-old infant. 2. Symptoms of biliary atresia would not be observable until several weeks of age. 3. Symptoms of Hirschsprung disease may be observable in the newborn nursery. 4. Umbilical hernia cannot be diagnosed at birth. 5. Diaphragmatic hernia will show symptoms immediately after birth due to compression of the lung.

14) Which is the priority nursing action for a premature neonate who is experiencing apnea? 1. Administering oxygen 2. Performing back blows and chest thrusts 3. Calling a code blue 4. Providing stimulation by stroking the back

Answer: 4 Explanation: 1. If the infant is not breathing, oxygen will not help. 2. This is intervention for choking, not apnea. 3. A code is not the initial response. If the nurse is unable to restart breathing, then a code should be initiated. 4. Tactile stimulation is often sufficient to restart the infant's respirations. Apnea of prematurity is due to immaturity of the respiratory center.

9) A nurse is preparing for the delivery of a newborn with a known diaphragmatic hernia defect. Which equipment should the nurse have on hand for the delivery? 1. Bag-valve-mask system 2. Sterile gauze and saline 3. Soft arm restraints 4. Endotracheal tube

Answer: 4 Explanation: 1. A bag-valve-mask system, or Ambu bag, could push air into the stomach and cause abdominal distension, increase pressure on the diaphragm, and impair breathing. 2. The defect is not external, so sterile gauze and saline are not needed. 3. Soft arm restraints might be necessary but at are not an immediate concern. 4. A diaphragmatic hernia (protrusion of abdominal contents into the chest cavity through a defect in the diaphragm) is a life-threatening condition. Intubation is required immediately so that the newborn's respiratory status can be stabilized.

13) Which food should the nurse remove from the food tray for a toddler-age client who is diagnosed with syndrome of inappropriate antidiuretic hormone (SIADH)? 1. Oatmeal 2. Yogurt 3. Biscuit 4. Watermelon

Answer: 4 Explanation: 1. A child with SIADH may have carbohydrates and fiber, such as in oatmeal. 2. A child with SIADH may have dairy products, such as yogurt. 3. A child with SIADH may have carbohydrates, such as in a biscuit. 4. A child with SIADH is on a fluid restriction. Watermelon contains significant fluid volume, so it would not be a good food for this child to consume.

17) Which rationale will the seasoned nurse share with the novice nurse regarding why the specific gravity for infants is lower than for older children? 1. The infant has a greater body surface area. 2. The infant has a higher basal metabolic rate. 3. The infant has a greater percentage of body weight that is water. 4. The infant's kidneys are less able to concentrate urine.

Answer: 4 Explanation: 1. Although this is true, it does not explain the lower specific gravity. 2. This statement is true but does not explain the specific gravity differences. 3. Although the statement is true, it does not explain the specific gravity differences. 4. This statement is accurate and explains why the specific gravity of the infant's urine is closer to water than an older child's urine specific gravity.

8) A nurse is planning care for a child with hyperkalemia. Which manifestation associated with the documented hyperkalemia requires immediate intervention by the nurse? 1. Hyperthermia 2. Respiratory distress 3. Seizures 4. Cardiac arrhythmias

Answer: 4 Explanation: 1. Excessive potassium is unrelated to the body temperature. 2. Potassium is needed for contractility of heart and skeletal muscles but not for the muscles of respiration. 3. Seizures are not an adverse outcome of hyperkalemia. 4. A child with hyperkalemia is at risk for cardiac problems that can be life threatening, such as arrhythmias.

10) Which action by the nurse is appropriate for a child who presents in the emergency department with an ankle injury? 1. Avoid compressing the area to allow tissue swelling as necessary. 2. Perform passive range-of-motion to the extremity. 3. Lower the extremity below the level of the heart. 4. Apply ice to the extremity.

Answer: 4 Explanation: 1. For the first 24 hours for a sprain, rest, ice, compression, and elevation (RICE) should be followed. Therefore, the nurse should apply a compression bandage to the extremity. 2. For the first 24 hours for a sprain, rest, ice, compression, and elevation (RICE) should be followed. Therefore, the nurse should rest the extremity rather than perform range-of-motion. 3. For the first 24 hours for a sprain, rest, ice, compression, and elevation (RICE) should be followed. Therefore, the nurse should elevate the extremity. 4. For the first 24 hours for a sprain, rest, ice, compression, and elevation (RICE) should be followed. Therefore, the nurse should apply ice to the extremity.

19) Which type of nutrition should the nurse include when planning care for a newborn who is diagnosed with galactosemia? 1. Goat's milk formula 2. Breast milk 3. Cow's milk-based formula 4. Lactose-free formula

Answer: 4 Explanation: 1. Goat's milk formula contains galactose and is excluded from the newborn's diet. 2. Breast milk contains galactose and is excluded from the newborn's diet. 3. Cow's milk-based formula contains galactose and is excluded from the newborn's diet. 4. A lactose-free formula is the type of nutrition the nurse should include in the teaching plan for this newborn.

7) The nurse is planning care for a school-age client who is postoperative for the surgical removal of the appendix. In addition to pharmacologic pain management, which should the nurse include in the plan of care to address pain? 1. Applying a warm, moist pack every 4 hours 2. Applying EMLA cream to the incision site prior to ambulation 3. Applying a cold, moist pack every 2 hours 4. Applying a pillow against the abdomen to splint the incision site when coughing

Answer: 4 Explanation: 1. Heat and moisture are not used on the incision area, as they can impair the healing process of the wound. 2. EMLA cream is a medication that requires a prescription. 3. Heat and ice are not used on the incision area, as they can impair the healing process of the wound. 4. A splint pillow placed on the abdomen is a nonpharmacologic strategy to decrease discomfort after an appendectomy.

5) Which immunization should the nurse include in a teaching session for parents of a toddler-age client to decrease the risk for epiglottitis? 1. Hepatitis B 2. Polio 3. Measles, mumps, and rubella (MMR) 4. Haemophilus influenzae type B (HIB)

Answer: 4 Explanation: 1. Hepatitis B, measles, mumps, rubella, and the polio virus are not causative agents for epiglottitis. 2. Hepatitis B, measles, mumps, rubella, and the polio virus are not causative agents for epiglottitis. 3. Hepatitis B, measles, mumps, rubella, and the polio virus are not causative agents for epiglottitis. 4. The Haemophilus influenzae type B (HIB) immunization can assist in prevention of epiglottitis.

18) Which prescription regarding an oral hydrocortisone for a toddler-age client diagnosed with congenital adrenal insufficiency should the nurse anticipate when the client is admitted to the hospital with pneumonia? 1. It will be discontinued. 2. It will be reduced. 3. It will be continued as previously prescribed. 4. It will be increased.

Answer: 4 Explanation: 1. Hydrocortisone is the glucocorticoid that helps the body deal with stress. It would be inappropriate to stop the medication. 2. The drug dosage would not be decreased. 3. During periods of stress, the child will need additional corticosteroids. 4. During periods of stress including illness and surgery, the dose of steroids needs to be increased. Page Ref: 1409

A child with nephrotic syndrome is placed on corticosteroids. About which side effects of corticosteroids should the nurse educate the family? a. Impaired balance b. Moon face c. Decreased appetite d. Hair loss

Answer: b. Moon face Feedback: Side effects of corticosteroids include moon face, increased hair growth, increased appetite, and mood swings. Impaired balance is not associated with corticosteroids.

9) Which parental statement indicates understanding of the process involved with a kidney transplant for a child with renal failure? 1. "We are happy our child will not have to take any more medicine after the transplant." 2. "We understand our child will not be at risk anymore for catching colds from other children at school." 3. "We will be glad we will not have to bring our child in to see the doctor again." 4. "We know it is important to see that our child takes prescribed medications after the transplant."

Answer: 4 Explanation: 1. Medications and general health promotion will be necessary. 2. The child will be on immunosuppressing drugs and will be at increased risk for colds and other illnesses. 3. Follow-up appointments will be necessary, as well as medications and general health promotion. 4. It is important that the nurse emphasizes compliance with treatments that will need to be followed after the transplant.

1) The nurse is providing care to a pediatric client, diagnosed with inflammatory bowel disease, who is prescribed daily prednisone. Which parental statement regarding administration of this drug indicates correct understanding of the teaching provided by the nurse? 1. "I will administer this medication between meals." 2. "I will administer this medication at bedtime." 3. "I will administer this medication one hour before meals." 4. "I will administer this medication with meals."

Answer: 4 Explanation: 1. Prednisone can cause gastric irritation and should not be given on an empty stomach. 2. Prednisone can cause gastric irritation and should not be given before bedtime on an empty stomach. 3. Prednisone can cause gastric irritation and should not be given on an empty stomach one hour before meals. 4. Prednisone, a corticosteroid, can cause gastric irritation. It should be administered with meals to reduce the gastric irritation.

8) Which assessment data would cause the nurse to suspect that a newborn requires further testing for cystic fibrosis? 1. Rectal prolapse 2. Constipation 3. Steatorrheic stools 4. Meconium ileus

Answer: 4 Explanation: 1. Rectal prolapse is a complication of the large, bulky fatty stools. 2. Constipation is not a symptom of cystic fibrosis. 3. Steatorrhea and rectal prolapse might be signs of cystic fibrosis seen in an older infant or child. 4. Newborns with cystic fibrosis might present in the first 48 hours with meconium ileus.

3) A child weighing 18.2 kg with a history of diabetes insipidus (DI) has been admitted to the hospital. Which healthcare provider prescription should the nurse question? 1. Stat electrolytes 2. Urine specific gravity with each void 3. DDAVP (desmopressin) PO 4. Restrict oral fluids to 500 mL every 24 hours

Answer: 4 Explanation: 1. Stat electrolytes would be an appropriate order to check for hypernatremia. 2. Urine specific gravity is checked because it is often low. 3. DDAVP is the drug of choice for a child with DI. 4. Fluid replacement, not fluid restriction, is necessary for child with DI.

4) The nurse is assessing abdominal girth for a pediatric client who presents with abdominal distension. Which nursing action is appropriate? 1. Measuring the girth just below the umbilicus 2. Measuring the girth just below the sternum 3. Measuring the girth just above the pubic bone 4. Measuring the girth around the portion of the stomach

Answer: 4 Explanation: 1. The circumference below the umbilicus would not be an accurate abdominal girth. 2. The circumference just below the sternum would not be an accurate abdominal girth. 3. The circumference just above the pubic bone would not be an accurate abdominal girth. 4. An abdominal girth should be taken around the largest circumference of the abdomen, just above the umbilicus.

18) Which is the priority nursing assessment when providing care for an infant at risk for dehydration? 1. Urine output 2. Urine specific gravity 3. Vital signs 4. Daily weight

Answer: 4 Explanation: 1. The infant is unable to concentrate urine and will continue to void dilute urine. Therefore, this is not the priority nursing assessment for an infant at risk for dehydration. 2. The infant's kidney is immature and unable to concentrate urine. Therefore, this is not the priority nursing assessment for an infant at risk for dehydration. 3. Pulse will elevate and blood pressure may drop, but the other vital sign findings will remain unchanged. However, this is not the best assessment of dehydration. 4. Daily weights on an infant provide the most accurate assessment of fluid balance.

2) A nurse is taking care of four different pediatric clients. Which child is at greatest risk for dehydration? 1. 7-year-old child with migraine headaches 2. 4-year-old child with a broken arm 3. 2-year-old child with cellulitis of the left leg 4. 18-month-old child with tachypnea

Answer: 4 Explanation: 1. The pediatric client with a chronic or acute condition that does not directly affect the GI or electrolyte system is at a lower risk than is a toddler with a condition that increases insensible water loss. 2. The pediatric client with an acute condition that does not directly affect electrolytes is at a lower risk than is a client with a condition that increases insensible water loss. 3. The pediatric client with an acute condition, such as a client with cellulitis that does not affect the GI or electrolyte system, is at a lower risk than is a toddler with a condition that increases insensible water loss. 4. The pediatric client with the greatest risk is under 2 years of age and with a condition that increases insensible fluid loss.

15) A school nurse suspects that a child who fell at recess has a fractured arm. Which should the nurse consider when applying a splint to transport the child to the hospital? 1. The splint is applied firmly enough to prevent swelling. 2. The arm is fully extended in the splint. 3. The splint is fully padded to prevent skin damage. 4. The joints above and below the suspected fracture are immobilized by the splint.

Answer: 4 Explanation: 1. The purpose of the splint is not to prevent swelling. 2. The nurse will not want to manipulate the arm, so the nurse will splint the arm in the position it is found. 3. The splint does not need to be padded. 4. This is the important concept in splinting—immobilizing the joint above and below the fracture to prevent movement of the bones.

16) Which is the priority nursing action for a child who presents in the emergency department after a motor vehicle accident with a sucking wound of the chest? 1. Placing the child in a Trendelenburg position 2. Beginning rescue breathing for the child 3. Beginning cardiac resuscitation for the child 4. Covering the child's wound with an air occlusive dressing

Answer: 4 Explanation: 1. This would not be the appropriate response to a sucking chest wound. 2. The child is conscious. Rescue breathing is not appropriate at this time. 3. There is no need for cardiac resuscitation at this time. 4. This prevents more air from entering the chest and is appropriate.

5) Which is the priority nursing diagnosis when planning care for a newborn who is born with esophageal atresia and tracheoesophageal fistula? 1. Ineffective Tissue Perfusion 2. Ineffective Infant Feeding Pattern 3. Acute Pain 4. Risk for Aspiration

Answer: 4 Explanation: 1. Tissue perfusion is not a primary problem with this condition. 2. The infant is always kept NPO (nothing by mouth) preoperatively, so ineffective feeding pattern would not apply. 3. Pain is not usually experienced preoperatively with this condition. 4. This is the most common type of esophageal atresia and tracheoesophageal fistula, where the upper segment of the esophagus ends in a blind pouch and a fistula connects the lower segment to the trachea. Preoperatively, there is a risk of aspiration of gastric secretions from the stomach into the trachea because of the fistula that connects the lower segment of the esophagus to the trachea.

16) Which clean-catch urinalysis finding should the nurse be most concerned for a child who is admitted to an urgent care center to rule out a urinary tract infection? 1. 2+ white blood cells 2. 1+ red blood cells 3. Urine appearance: cloudy 4. Specific gravity: 1.009

Answer: 4 Explanation: 1. White blood cells are expected. 2. Red blood cells are common in the urine of a child with a urinary tract infection. 3. With white blood cells in the urine, this is a common finding. 4. This is a very dilute urine. With white blood cells (WBCs), red blood cells (RBCs), and bacteria in the urine, you would expect the urine to contain more solutes.

19) Which clinical manifestations should the nurse expect when assessing a pediatric client who is diagnosed with Legg-Calvé-Perthes disease? Select all that apply. 1. Limited abduction of the affected hip 2. Asymmetry of thigh fat folds 3. Telescoping of the thigh 4. Muscle weakness 5. Atrophy of the muscles

Answer: 4, 5 Explanation: 1. Limited abduction of the affected hip is a clinical manifestation associated with clinical hip dysplasia, not Legg-Calvé-Perthes disease. 2. Asymmetry of the thigh fat folds is a clinical manifestation associated with clinical hip dysplasia, not Legg-Calvé-Perthes disease. 3. Telescoping of the thigh is a clinical manifestation associated with clinical hip dysplasia, not Legg-Calvé-Perthes disease. 4. Muscle weakness is an expected clinical manifestation associated with Legg-Calvé-Perthes disease. 5. Atrophy of the muscles is not an expected clinical manifestation associated with Legg-Calvé-Perthes disease.

21) Which interventions should the nurse include in the plan of care for a pediatric client who is receiving peritoneal dialysis in the treatment of chronic renal failure to prevent infection? Select all that apply. 1. Provide small, frequent meals. 2. Avoid battles over nutritional intake. 3. Administer supplements by tube feedings, if needed. 4. Implement hand hygiene frequently. 5. Perform daily catheter site care.

Answer: 4, 5 Explanation: 1. This intervention is appropriate to meet the child's nutritional needs; however, this will not prevent infection. 2. This intervention is appropriate to meet the child's nutritional needs; however, this will not prevent infection. 3. This intervention is appropriate to meet the child's nutritional needs; however, this will not prevent infection. 4. Aseptic technique reduces chance of introducing bacteria into the abdomen. 5. Skin around the catheter site will have fewer organisms that could potentially cause infection.

The nurse takes care of a newborn diagnosed with Eagle-Barrett syndrome. Which comment by the parent indicates teaching was effective? a. "As a teen, my child might develop end-stage renal disease." b. "My infant has about 3 months to live due to severe renal problems." c. "The skin of his bottom looks like a prune due to poor peristalsis." d. "He has this syndrome from a recessive gene; my next baby will have it too

Answer: a. "As a teen, my child might develop end-stage renal disease." Feedback: Children with Eagle-Barrett syndrome (prune belly) will develop end-stage renal disease in childhood or adolescence because of inadequate renal function. The skin covering the abdominal wall is thin and resembles a wrinkled prune. Death occurs in the neonatal period due to pulmonary hypoplasia and severe renal dysfunction. Prune belly syndrome is thought to be related to a fetal urinary tract obstruction or a specific injury, not genetics

The mother of a 6-year-old calls the clinic because her child is wetting the bed. Which assessment question by the nurse is most important? a. "Is there a family history of renal or urinary problems?" b. "What happens when the child wets the bed?" c. "At what age was the child potty-trained?" d. "How is the child doing in school?"

Answer: a. "Is there a family history of renal or urinary problems?" Feedback: Enuresis more often occurs in children who have a positive family history, so the primary assessment question is to determine whether there is a family history. The other questions are important to ask when assessing a child with enuresis, but are not the priority.

Following a motor vehicle accident and successful cardiopulmonary resuscitation, arterial blood gases are drawn from a 13-year-old client. What will the nurse identify as the result of this test? a. Acid-base balance b. Prognosis c. Capillary metabolic exchange d. Carbonic acid level

Answer: a. Acid-base balance Feedback: The test provides information about immediate status of the client's acid-base balance. It will not provide data that can predict future outcomes. Capillary blood gases are done on newborns and infants to decrease the amount of blood used, but that is not the test referred to in this question. Carbonic acid contributes to the acid-base balance, but is not measured specifically in this test as a percentage.

Assessment of a 2-year-old by a nurse in the emergency department reveals the following: edema, hematuria, hypertension, and oliguria. What would the nurse assess as the most likely cause of these symptoms? a. Acute renal failure b. Urinary tract infection c. Vesicoureteral reflux d. Bladder exstrophy

Answer: a. Acute renal failure Feedback: There are several things that can cause acute renal failure, including hemolytic uremic syndrome, nephritic syndrome, and severe dehydration. Most children with acute renal failure are admitted to a pediatric intensive care unit. A urinary tract infection would not cause any of the listed symptoms. Bladder exstrophy is a congenital defect discovered at birth. Vesicoureteral reflux is a backflow of urine from the bladder to the kidneys.

In addition to an impaired gas exchange, which other diagnosis will a child in the early stages often have as well? a. Anxiety related to hypoxia b. Fatigue related to air trapping c. Injury related to fatigue and dehydration d. Delayed Development related to hypoxia

Answer: a. Anxiety related to hypoxia Feedback: Air trapping is not present in all cases of impaired gas exchange. Delayed development does not occur unless the condition is chronic or acutely damaging. The early phase of impaired gas exchange does not cause injury or dehydration, although fatigue can occur.

Which instructions would the nurse provide to the family of a child who has undergone a hypospadias repair? (Select all that apply.) a. Avoid tub baths until the catheter is removed. b. Notify the primary healthcare provider if there is blood in the urine. c. The child should avoid the straddling position with play. d. It is important that the catheter be left in place. e. Notify the primary healthcare provider if the child goes more than 30 minutes without urine output.

Answer: a. Avoid tub baths until the catheter is removed; c. The child should avoid the straddling position with play; d. It is important that the catheter be left in place. Feedback: The nurse should discuss with the family the importance of leaving the catheter in place, notifying the primary healthcare provider if the child goes more than 1 hour without urine output, notifying the primary healthcare provider if the child avoids the straddling position with play, and avoiding baths until the catheter is removed. It is normal to see blood-tinged urine for several days after surgery.

For what condition does the nurse taking care of a 5-year-old newly diagnosed with Crohn disease teach the parents that their son may be at risk later? a. Cancer b. Malabsorption c. Atresia d. Hepatitis

Answer: a. Cancer Feedback: The risk of cancer is greatly increased for the child diagnosed with Crohn disease. Symptoms of Crohn disease include cramped abdominals followed by diarrhea, fever, anorexia, growth failure or weight loss, general malaise, and joint pain. The risks for malabsorption, atresia, and hepatitis do not increase in clients with Crohn disease.

A community health nurse is educating a high school class about sexually transmitted infections (STIs). Which information should be included in the presentation? a. Chlamydia can be asymptomatic. b. Ejaculation must occur for gonorrhea to be transmitted. c. A condom will protect teenagers from getting herpes. d. Intercourse is the only means of transmitting STIs.

Answer: a. Chlamydia can be asymptomatic. Feedback: Abstinence from all forms of sexual contact will protect a teenager from getting an STI. Many people have Chlamydia without knowing it, as it can be asymptomatic. A condom does not always protect a teenager from getting herpes, because the herpes lesion might not be covered by the condom and the condom might break. Gonorrhea can be transmitted without ejaculation.

The mother of a 2-year-old calls the clinic nurse in a panic, stating, "I think my child swallowed a marble!" Which signs does the nurse know are indicative of a foreign-body aspiration? a. Coughing and dysphonia b. Fear and wheezing c. Hypoxia and choking d. Nasal flaring and crying

Answer: a. Coughing and dysphonia Feedback: Clinical manifestations of foreign body aspiration include a sudden onset of choking, spasmodic coughing, shortness of breath, or dysphonia. Fear, wheezing, hypoxia and nasal flaring are later signs of progressing respiratory distress.

Which intervention by the nurse is most important when taking care of a child with severe dehydration? a. Monitor weight daily. b. Monitor for crackles in the lungs. c. Monitor level of consciousness. d. Monitor serum sodium levels.

Answer: a. Monitor weight daily. Feedback: When managing a child with severe dehydration, the nurse must weigh the child daily with the same scale and without clothing to compare past weights and calculate weight loss. A dehydrated child will not have crackles in the lungs. Level of consciousness and serum sodium levels will be monitored, but the priority is hydration.

A 12-year-old is being treated for acute respiratory distress syndrome. Which assessment finding would be indicative of the nursing diagnosis Impaired Gas Exchange? a. Oxygen saturation of 62% b. Heart rate of 100 bpm c. Respiratory rate of 30/min d. Bicarbonate level of 38

Answer: a. Oxygen saturation of 62% Feedback: The incorrect options do not contain evidence of abnormal gas exchange values. Pallor, tachycardia, hypertension, and fever can occur with impaired gas exchange but alone do not yield that nursing diagnosis. Bradycardia, lethargy, appearing flushed, and hypothermia could be true in unusual circumstances but are not the typical picture of Impaired Gas Exchange. Elevated bicarbonate, metabolic alkalosis, irritability, and pallor do not reflect gas exchange abnormalities.

Which intervention would not be included in the preoperative plan of care for an infant with an omphalocele? a. Push the exposed abdominal contents back into the abdomen. b. Administer intravenous fluids. c. Assess for signs of other congenital anomalies. d. Care for the infant in a radiant warmer.

Answer: a. Push the exposed abdominal contents back into the abdomen. Feedback: Care of an infant with an omphalocele (congenital malformation where abdominal contents herniate through the umbilical cord covered by a translucent sac) is aimed at protection of abdominal contents. Aggressive attempts at replacing abdominal contents can lead to numerous problems, including increased abdominal pressure, impaired respiratory status, and bowel perforation. The goals should be to protect the infant from hypothermia, replace fluids, prevent infection, and look for other associated anomalies.

Which intervention would be appropriate when a nurse is caring for a child with acute postinfectious glomerulonephritis (APIGN)? a. Screen family members for strep throat. b. Offer a high-protein diet. c. Maintain strict fluid restriction. d. Monitor the child for hyperactivity.

Answer: a. Screen family members for strep throat. Feedback: Rationale: The child with APIGN should have a diet low in protein with no added salt. Family members should be checked for strep throat, and the child should be monitored for any neurological changes.

The nurse is monitoring a 6-year-old admitted to the emergency department with sixth disease. For which medical emergency does the nurse watch closely? a. Seizure activity b. Increased pain c. Excessive diarrhea d. Sore throat

Answer: a. Seizure activity Feedback: Febrile seizures are a complication of sixth disease and the nurse should watch the client closely. Pain, diarrhea, and sore throat are not symptoms or complications of sixth disease.

A 10-year-old child presents to the emergency department with decreased urinary output, lethargy, and confusion. The nurse suspects hypernatremia. About what condition is the nurse most concerned and how can the hypernatremia be treated? a. Seizures/hypotonic fluid b. Coma/hypertonic fluid c. Confusion/salt tablets d. Anuresis/tap water

Answer: a. Seizures/hypotonic fluid Feedback: Seizures can occur when hypernatremia occurs rapidly or is severe. Severe hypernatremia can be fatal. Hypernatremia is treated by intravenous administration of hypotonic fluid. A decreased level of consciousness manifested by confusion, lethargy, or coma can result from shrinking of the brain cells; anuresis may also occur, but the treatments associated with these conditions will be a hypotonic fluid.

Which tasks should the nurse perform rather than delegate to an assistant? (Select all that apply.) a. Suctioning a 2-year-old with a tracheostomy b. Changing the diaper of the 3-month-old infant recovering from RSV c. Walking with a 2-year-old who has an IV receiving antibiotics for pneumonia d. Relieving the nurse who is watching a 2-year-old with croup, because he now sounds quiet e. Taking the temperature of an 8-month-old infant with bronchiolitis whose respirations are 68 and who is irritable

Answer: a. Suctioning a 2-year-old with a tracheostomy; d. Relieving the nurse who is watching a 2-year-old with croup, because he now sounds quiet; e. Taking the temperature of an 8-month-old infant with bronchiolitis whose respirations are 68 and who is irritable Feedback: Respirations of 68 are high for an 8-month-old infant. The nurse needs to assess for retractions and wheezing. A 2-year-old who becomes quiet following respiratory distress could be experiencing decompensation and requires an evaluation. Suctioning is a sterile procedure that only the nurse should perform.

A 4-year-old child with croup is brought to the emergency department. The child is anxious and crying and has a high-pitched stridor, retractions, and a barky cough. After administration of cool mist therapy, which assessment finding would indicate significant improvement in the child's respiratory status? a. The child is less anxious. b. The respiratory rate is decreased. c. Wheezing is less loud. d. The child drinks 8 ounces of fluid.

Answer: a. The child is less anxious. Feedback: All responses indicate conditions that are beneficial to the child. Respiratory distress and hypoxia cause anxiety as this vital life function is threatened. When anxiety improves, the nurse knows that the respiratory status must be improving as well, even if signs and symptoms continue.

Which statement by the parent of an uncircumcised male infant would indicate the need for further teaching? a. "Frequent diaper changes are important." b. "I should forcibly retract the foreskin once a day." c. "Once the foreskin is retractable, it should be returned to its normal position after cleaning." d. "Harsh soaps should be avoided."

Answer: b. "I should forcibly retract the foreskin once a day." Feedback: The parent should never force the foreskin to retract, due to the fact that it may cause paraphimosis, which is where the foreskin cannot be returned to its normal position. Harsh soaps should be avoided. Frequent diaper changes are important to prevent irritation, and once the foreskin is retractable in early childhood, always return it to its normal position after cleaning.

The nurse is caring for a group of infants in the neonatal intensive care unit. Which infant would require preparation for immediate surgery due to risk of life-threatening respiratory distress? a. An infant with an umbilical hernia b. An infant with a diaphragmatic hernia c. An infant with a cleft palate d. An infant with gastroesophageal reflux

Answer: b. An infant with a diaphragmatic hernia Feedback: Gastroesophageal reflux, cleft palate, and umbilical hernia do not cause respiratory distress and are not considered surgical emergencies. A diaphragmatic hernia will cause the abdominal organs to extend into the chest, causing pressure on the thoracic cavity. Only 50% of afflicted infants survive.

In obtaining a nursing history on an 18-month-old with diarrhea, which questions might help to identify the cause of the problem? (Select all that apply.) a. Has the child taken diphenhydramine in the past week? b. Do any other family members have diarrhea? c. Has the child been on antibiotics recently? d. Does the child have any food sensitivities? e. Has the child traveled recently?

Answer: b. Do any other family members have diarrhea?; c. Has the child been on antibiotics recently?; d. Does the child have any food sensitivities?; e. Has the child traveled recently? Feedback: A complete history of the child with diarrhea is important to finding the cause. Questions should cover recent travel, medication use, exposures, and foods eaten. Diphenhydramine is an antihistamine that does not cause diarrhea. Similar symptoms in other family members suggest infectious etiology.

A 6-month-old infant is admitted with severe dehydration. Effectiveness of therapy is evaluated with which assessment measures? (Select all that apply.) a. Documenting abdominal girth every shift b. Documenting mucous membrane moisture every shift c. Daily weights each day on a rotating shift d. Recording intake and output accurately e. Evaluating level of consciousness continuously

Answer: b. Documenting mucous membrane moisture every shift; d. Recording intake and output accurately; e. Evaluating level of consciousness continuously Feedback: All of the choices represent assessment measures that evaluate the effectiveness of therapy except abdominal girth, which does not provide information regarding hydration status, and daily weights on a rotating schedule. Daily weights should be done, but they must be completed on the same scale at the same time each day while the infant is wearing no clothing.

A school nurse teaches a coaching staff about heat-related illnesses. Which action by a coach indicates to the nurse that teaching was effective? a. Sleep 2-3 hours in the middle of the day during all-day practice. b. Have cell phones or other mechanisms to call for emergency assistance. c. During activity, stop for fluids every 30 minutes. d. Allow 1-2 hours' rest during the middle of the day, with fluids and food provided.

Answer: b. Have cell phones or other mechanisms to call for emergency assistance. Feedback: Rest 2-3 hours during the day of all-day practice, but sleep is not required. Stop for fluids every 15-20 minutes. It is essential to have a means of communication in case of emergency.

Which issue is important to discuss when educating a family about nocturnal enuresis? a. Limit daytime fluids. b. Have the child double-void before going to bed. c. Administer laxatives daily. d. Refer the child to counseling immediately.

Answer: b. Have the child double-void before going to bed. Feedback: Counseling is not always indicated. Promoting regular stools and having the child double-void before bed are appropriate interventions. Limiting daytime fluids has not been shown to be effective.

The nurse is preparing a pediatric client for a barium enema. Which diagnosis would support the need for this diagnostic test? a. Gastroschisis b. Intussusception c. Appendicitis d. Pyloric stenosis

Answer: b. Intussusception Feedback: Intussusception occurs when the intestine invaginates into another, causing pain with vomiting and passage of brown stool. The stools eventually can resemble currant jelly. Pyloric stenosis is a stenosis between the stomach and duodenum. Gastroschisis is a congenital defect where there is herniation of abdominal contents outside the abdominal wall. Appendicitis is an inflammatory process of the appendix

An 8-year-old is admitted to the emergency department with an injury to the abdomen with single organ involvement. Which type of injury does the nurse suspect? a. High-velocity blunt trauma b. Sports-related trauma c. Penetrating trauma d. Bike-related trauma

Answer: b. Sports-related trauma Feedback: Sports-related abdominal trauma is often associated with a direct blow to the abdomen, and a single organ is usually injured. High-velocity blunt trauma usually involves multiple organs. Blunt trauma may not be apparent in penetrating traumas and would have to be assessed to determine what injury lies beneath the skin surface. Bike-related traumas can result in serious abdominal injuries.

Which condition in males would the nurse assess as a medical emergency? a. Cryptorchidism b. Testicular torsion c. Phimosis d. Inguinal hernia

Answer: b. Testicular torsion Feedback: Testicular torsion is a medical emergency and should be surgically repaired within 4-6 hours of onset. The testis rotates on its spermatic cord, obstructing blood supply. Inguinal hernia is when a portion of abdominal cavity protrudes into the groin. It is usually repaired after 3 months of age and is not considered emergent unless the hernia is incarcerated. Cryptorchidism is when a testicle is not descended. It is present at birth, and if the testicle does not descend by 1-2 years of age, it is repaired. Phimosis is when the skin around the glans of the penis is not retractable by young childhood.

A nurse is evaluating a parent performing a clean intermittent catheterization on a pediatric client. What would be an indication that the parent needs additional teaching? a. The parent uses a size 4 catheter for the procedure. b. The parent states that the child should be awakened once during the night to be catheterized. c. The parent uses a water-soluble lubricant to coat the end of the catheter. d. The parent uses gentle pressure to advance the catheter if resistance is met.

Answer: b. The parent states that the child should be awakened once during the night to be catheterized. Feedback: It is necessary to perform intermittent catheterization every 3-4 hours but not while the child is sleeping at night. A size 4 or 5 catheter is used for the procedure. A water-soluble lubricant, not Vaseline, is used. In males, the sphincter muscle located at the entrance to the bladder will cause resistance to the catheter, but with gentle pressure, the catheter will advance into the bladder.

The nurse notes several changes in the past 24 hours in a child with heart failure. Which finding is the most significant in assessing the child for fluid volume overload? a. Presence of lung crackles b. Weight gain of .4 kg c. Bounding pulse d. Jugular venous distention

Answer: b. Weight gain of .4 kg Feedback: All are signs of fluid volume overload. However, an increase of weight is always the best indicator of an increase in fluid. The other options indicate heart failure as a result of the increase in fluid.

Which instruction should be provided to the parents of an infant with gastroesophageal reflux? a. "Feed every 4-5 hours to prevent overfeeding." b. "Place in a seated position for 10 minutes after feedings." c. "Elevate the head of the crib at all times." d. "Burp every 3-4 ounces with feeding."

Answer: c. "Elevate the head of the crib at all times." Feedback: Management of gastroesophageal reflux includes administering small, frequent feedings and burping every 1-2 ounces. Elevating the head of the bed and holding the infant upright for 30 minutes after feeding help minimize the reflux. Putting the infant in a seated position can increase the pressure on the abdomen, causing reflux to increase.

A school nurse initiates an asthma action plan after checking a student's peak expiratory flow averages after three readings. Which peak expiratory flow average indicates that no action be taken? a. 35% b. 65% c. 85% d. 40%

Answer: c. 85% Feedback: A reading of 80% to 100% of peak expiratory flow is green, or best. A reading of 50% to 80% is yellow, or a warning. In order to prevent the symptoms from increasing, action must be taken. Less than 50% is red, or a warning. This emergency requires medical care.

Which client would the nurse suspect to have pyloric stenosis? a. A 7-month-old with choking episodes b. An 11-year-old with an olive-shaped abdominal mass c. A 5-week-old infant with projectile vomiting d. A 2-year-old with a harsh cough

Answer: c. A 5-week-old infant with projectile vomiting Feedback: The most likely incidence of pyloric stenosis is in a 2- to 8-week-old infant. The common symptoms are nonbilious projectile vomiting, irritability, and failure to gain weight.

In which child does the nurse anticipate a potential respiratory arrest following an assessment? a. A 5-month-old infant with RSV who is sleeping and has a respiratory rate of 24 b. A 2-year-old with epiglottitis who was intubated in the emergency department c. A 6-year-old with asthma who was previously wheezing and now has decreased breath sounds d. A 4-year-old, status post-tension pneumothorax from a motor vehicle accident with a chest tube in place, who complains of pain

Answer: c. A 6-year-old with asthma who was previously wheezing and now has decreased breath sounds Feedback: All of the children are acutely ill. A child with asthma who was wheezing and now has decreased breath sounds is acutely ill. This child's ability to move air is decreasing and is approaching respiratory arrest. Intubation protects the airway from closing in epiglottitis and a chest tube is the treatment for tension pneumothorax in a different room; therefore, these children are stable. The infant with RSV is sleeping with a normal respiratory rate, so there is no immediate danger here.

Which comments by the parents of a 7-year-old child with asthma indicate comprehension of instructions regarding medication use for control of the illness? a. The medications are too complicated for a 7-year-old to understand. b. If a spacer is used, a whistling sound indicates that the medication is being inhaled correctly. c. A spacer used on an inhaler helps trap the medication so it is inhaled more readily. d. Dry powder inhalers are for adult use only.

Answer: c. A spacer used on an inhaler helps trap the medication so it is inhaled more readily. Feedback: A 7-year-old is at an age when medication administration responsibility ought to be initiated. The spacer whistle is significant, although its significance varies with each type of spacer. Children may use dry powder inhalers when they are old enough to have a rapid inhalation.

Which intervention would the nurse include in the care of an infant following surgical repair of a cleft lip? a. Let the infant touch the suture lines as a means of self-comforting. b. Position the infant in the supine position for feedings to avoid aspiration. c. Administer pain medications as ordered. d. Use a special feeding device with shorter nipples.

Answer: c. Administer pain medications as ordered. Feedback: Special feeding devices with long nipples usually are used, and the infant is fed in the sitting position to avoid aspiration. Some soft restraints may be used to prevent the infant from touching the suture line.

A nurse is caring for an 11-month-old infant admitted for watery, green diarrhea; vomiting; and fever. He is diagnosed with gastroenteritis with no known source at this time. Which nursing diagnosis should be the highest priority? a. Altered Nutrition b. Anxiety related to hospitalization c. Fluid Volume and Electrolyte Imbalance d. Altered Family Coping

Answer: c. Fluid Volume and Electrolyte Imbalance Feedback: Fluid and electrolyte imbalance is a safety issue and a potentially life-threatening event. Although all of the diagnoses should be addressed, this takes precedence.

A nurse is discharging an infant after a pyloric stenosis repair. Which statement by the mother would indicate the need for further instructions prior to discharge? a. "I should call the doctor if my infant's temperature rises above 101 degrees." b. "I should fold the diaper down so it does not irritate the incision." c. "My infant's incision will need to be observed for redness, swelling, or discharge." d. "If my infant vomits, I should hold feedings for 6 hours."

Answer: d. "If my infant vomits, I should hold feedings for 6 hours." Feedback: It is normal for an infant to vomit occasionally after having surgery for pyloric stenosis. The infant should be fed on a normal feeding schedule. All other statements about checking the incision site, folding the diaper, and calling the doctor if there is a fever are true.

Which assessment finding would lead the nurse to suspect esophageal atresia in an infant? a. Hypotonicity b. Excessive crying c. Abdominal distention d. Excessive drooling

Answer: d. Excessive drooling Feedback: The classic symptoms in an infant with esophageal atresia are excessive drooling often accompanied by cyanosis, choking, and coughing. Low blood pressure, excessive crying, and hypotonicity are not common signs of esophageal atresia.

A child is 24 hours postoperative following major trauma, and has received a total of eight units of packed red blood cells during the perioperative period. The child is flaccid, and has diarrhea and peaked T-waves on the electrocardiogram. About which electrolyte abnormality would the nurse call the primary healthcare provider to obtain an electrolyte panel? a. Hypercalcemia b. Hypernatremia c. Hypermagnesemia d. Hyperkalemia

Answer: d. Hyperkalemia Feedback: Hypernatremia is associated with dehydration and thirst. Hypercalcemia causes neuromuscular depression and constipation. Etiology is related to malignancy, overintake, and parathyroid disorder. Hypermagnesemia symptoms are similar to hyperkalemia, but the etiology is different. Hypermagnesemia is due to renal failure or administration. This client could be experiencing renal failure but that information is not given.

A nurse is assessing a 3-year-old for hemolytic uremic syndrome (HUS). Which assessment finding would be most characteristic of HUS? a. Fever b. Severe cough c. Diarrhea d. Oliguria

Answer: d. Oliguria Feedback: HUS is characterized by the classic triad of symptoms: thrombocytopenia, hemolytic anemia, and acute renal failure. Severe cough, fever, or diarrhea alone is not a sign of HUS. The problem usually is preceded by a urinary tract infection, upper respiratory infection, or acute gastroenteritis 1-2 weeks prior to the HUS.

Arterial blood gases results indicate pH 7.33 and PCO2 of 38 mmHg following arrest and subsequent resuscitation of a 3-year-old child. Which nursing intervention should be utilized to attempt to correct this metabolic disorder? a. Assess the effectiveness of the respiratory pattern. b. Determine whether the endotracheal tube is positioned correctly. c. Administer sodium bicarbonate 1 mEq/kg IV. d. Treat the cause of the acidosis.

Answer: d. Treat the cause of the acidosis. Feedback: Sodium bicarbonate is used to correct serious metabolic acidosis. Metabolic acidosis is present, but sodium bicarbonate is given only in serious imbalance. We do not have enough information to determine whether that is the case. The medication often complicates acid-base imbalance. Airway always is important to assess, but the PCO2 level indicates that respirations are not contributing to the problem. The best answer is to find and treat the cause of the acidosis.

10) Which teaching point should the nurse include when providing education to an adolescent client, who participates in soccer, regarding the plan of care for diabetes mellitus? 1. Decreased food intake 2. Increased doses of insulin 3. Increased food intake 4. Decreased doses of insulin

Explanation: 1. Decreased food intake would increase the chance of hypoglycemia. 2. Increased dose of insulin would cause hypoglycemia. Exercise causes insulin to be used more efficiently by the body, so an increase in insulin would not be needed. 3. An increase in physical activity requires an increase in caloric intake to prevent hypoglycemia. 4. A decreased dose of insulin would not allow the sugar to enter the cells where it is needed during exercise. Page Ref: 1421


Conjuntos de estudio relacionados

APES Chapter 15: Air Pollution and Stratospheric Ozone Depletion

View Set

How Things Work Practice Problems 1-2

View Set

Nursing Care of Children ATI Practice Quiz One

View Set